You are on page 1of 117

1.

What is the difference between asteroids Answer : (b)


and comets?

1. Asteroids are small rocky planetoids, while


comets are formed of frozen gases held
together by rocky and metallic material

2. Asteroids are found mostly between the orbits


of Jupiter and Mars, while Comets are found
mostly between Venus and Mercury

3. Comets show a perceptible glowing tail, while


asteroids do not

a) 1 and 2 b) 1 and 3

c) Only 3 d) All of these

2. Which of the following statements is the Answer : (d)


reason for ‘Seasons’ on the surface of the
Earth?

a) Rotation of the Earth

b) Revolution of the Earth around the Sun

c) The 23.5 degree inclination of the Earth’s


rotation axis with respect to the plane of the
ecliptic

d) Revolution of Earth in elliptical orbit around


the Sun

3. The greatest seasonal contrast of the Answer : (d)


insolation on the Earth is, in which of the
following latitudinal zones?

a) Equatorial

b) Tropical

c) Temperate

d) Polar

1
4. When the winds blow from all sides to the Answer : (c)
centre low in an anti-clockwise direction, then
this phenomenon is known as

a) anti-tropical cyclones of Southern hemisphere

b) temperate cyclones of Northern hemisphere

c) tropical cyclones of Northern hemisphere

d) tropical cyclones of Southern hemisphere

5. Nearly 30% of the solar radiations return back Answer : (c)


to the space without contributing anything to
the Earth’s surface temperature. This amount
of radiation is known as

a) black body

b) tropopause

c) Earth’s albedo

d) Mesopause

6. We always see the same face of the Moon, Answer : (c)


because

a) it is smaller than the Earth

b) it revolves on its axis in a direction

c) it takes equal time for revolution around the


Earth and rotation on its own axis

d) it rotates at the same speed as the Earth


around the Sun

2
7. Which one among the following statements Answer : (c)
about the International Date Line is not
correct?

a) The International Date Line is largely based on


the 180 meridian

b) The difference in time between the places just


either side of the International Date Line is
almost day

c) The difference in time to the extent of one day


on either side of the International Date Line is
caused by inclined axis of the Earth

d) The International Date Line mostly passes


through the Pacific Ocean

8. Which part of the Earth’s Surface experience Answer : (b)


least variation in incoming solar radiation
throughout the year?

a) Poles

b) Equatorial regions

c) Tropics of cancer and Capricorn

d) Arctic and Antarctic circles

9. Which one of the following represents the Answer : (b)


inner planets?

a) Planets between the Sun and the Earth

b) Planets between the Sun and the Belt of


asteroids

c) Planets in gaseous state

d) Planets without satellite

3
10. Consider the following statements Answer : (d)
1) When the sun, the earth and the moon are
aligned in a straight line, then position is
referred to as SYZYGY
2) When the sun and moon are in straight line it
results in spring tide
3) When the sun and moon are in right angle it
results in neap tide
Which of the statements given above are correct
a) 1 and 2
b) 2 and 3
c) 1 and 3
d) 1,2 and 3

11. Which one of the following statements Answer : (c)


regarding Chinook winds is not correct?

a) These are local winds

b) They descend along the eastern slopes of the


rocky mountain in mid latitudes

c) They bring lots of rain with them

d) They benefit agriculture

12. Which one of the following statements is not Answer : (b)


correct?

a) The Sial and Sima together form the Earth’s


crust

b) Sima is lighter than Sial

c) Mantle is composed mainly of very dense


rocks, rich in olivine

d) Core is made up of mainly of Iron with some


Nickel and is called Nife

4
Answer : (d)
13. Which one of the planets is known as morning
star as well as evening star because it is the 1st
to appear in the evening and the last to
disappear in the morning?

a) Jupiter

b) Mars

c) Mercury

d) Venus

Answer : (b)
14. Which one of the following pairs is correctly
matched?

Geographical feature Region

a) Abyssinian plateau Arabia

b) Atlas mountains North-Western Africa

c) Guiana Highlands South-Western Africa

d) Okavango Basin Patagonia

Answer : (a)
15. The latitude is the angular distance of a point
of the earth’s surface, north or south of the
equator as measured from the

a) centre of the Earth

b) Tropics of cancer and Capricorn

c) Equator

d) Poles

5
16. Consider the following statements Answer : (a)

1) International Date line is drawn zigzag to avoid


landmass

2) International date line is 180 degree west as


180 degree east of Greenwich

3) A ship sailing Westward from Greenwich when


crossing international; date line would put
back the date by a day

Which of the statements given above are correct

a) 1 and 2

b) 1 and 3

c) 1,2 and 3

d) Only 3

17. How much of a time period 15 degree of Answer : (d)


longitudes account for?

a) 4 minutes

b) 15 minutes

c) 30 minutes

d) 60 minutes

18. Which of the following is not correctly


matched?

a) Prime Meridian 0 degree

b) International Date Line 180 degree

c) Arctic circle 66.5 degree north

d) Tropics of cancer 23.5 degree south

6
19. Consider the following statements Answer : (c)

1) Jupiter is the largest planet in the solar system,


is diameter 109 times larger than Earth’s

2) Jupiter’s four large moons called the Galilean


moons

Which of the following statements given above is/are


correct?

a) Only 1 b) Only 2

c) Both 1 and 2 d) None of these

20. In the structure of planet earth, below the Answer : (c)


mantle, the core is mainly made up of which
one of the following?

a) Aluminium

b) Chromium

c) Iron

d) Silicon

21. Which is the 2nd most abundant metal in the Answer : (b)
earth’s crust?

a) Iron

b) Aluminium

c) Copper

d) Zinc

7
22. Which scale is used to measure the magnitude Answer : (a)
of earthquake?

a) Richter

b) Metric

c) Centigrade

d) Newton

23. What is the process that tends to build up the Answer : (c)
land surface by the deposition of solid material
is its lower areas, known as?

a) Abrasion

b) Agglomeration

c) Aggradation

d) Attrition

24. Which one of following African countries is not Answer : (a)


landlocked?

a) Benin

b) Chad

c) Lesotho

d) Mali

8
25. Match the following Answer : (a)

List-I (Fishing bank) List-II (Coast)


A) Dogger bank 1) New Foundland coast
B) Grand Bank 2) Australia coast
C) Great Barrier Reef 3) North Sea
D) Tonga Trench 4) East of Fiji island
Codes
A B C D
a) 3 1 2 4
b) 4 2 1 3
c) 3 2 1 4
d) 4 1 2 3

26. Match the following Answer : (d)

List 1 List 2
(Volcano) (Country)

A) Mount Etna 1) India

B) Kilimanjaro 2) USA

C) Katmai 3) Tanzania

D) Barren Island 4) Italy

Codes
A B C D
a) 1 3 2 4
b) 4 2 3 1
c) 1 2 3 4
d) 4 3 2 1

27. The shattering and breaking up of the stream Answer : (c)


load through collisions and mutual abrasion,
leading to rounding of pebbles and boulders is
called as
a) Cavitation
b) Corrasion
c) Attrition
d) Solution

9
28. Which of the following rocks can undergo Answer : (a)
folding?

1) Sedimentary

2) Metamorphic

3) Igneous

Select the correct answer using the codes given below

a) only 1

b) 1 and 2

c) 1 and 3

d) All of these

29. Which of the following is true about river Answer : (d)


deposition?

1) Deposition takes place when a river has


insufficient energy to transport its load.

2) Larger particles are deposited first.

3) Deposition will occur when the speed of a river


is reduced.

Select the correct answer using the codes given below:

a) 1 and 3 b) 2 and 3

c) 1 and 2 d) All of these

30. Which one of the following is produced by rain Answer : (c)


water action?

a) Gorge

b) Cliff

c) Gully

10
d) Dome

31. Mid-Atlantic Ridge is an example of Answer : (a)

a) constructive plate margin

b) destructive plate margin

c) conservative plate margin

d) None of the above

32. Among the following sedimentary rocks, which Answer : (b)


one is of organic origin?

a) Gypsum

b) Limestone

c) Nitre

d) Rock Salt

33. Folding is the result of Answer : (c)

a) epeirogenetic force

b) coriolis force

c) orogenetic force

d) exogenetic force

11
34. Which one of the following processes of Answer : (a)
weathering belongs to both mechanical and
chemical weathering?

a) Crystallisation

b) Exfoliation

c) Hydration

d) Carbonation

35. Consider the following statements Answer : (c)

1) Up folded rock beds are called Anticlines

2) Down folded rock beds are called Synclines

Which of the above statements are not true?

a) only 1

b) only 2

c) Both 1 and 2

d) Neither 1 nor 2

36. Radioactive decay provides an internal source Answer : (a)


of heat for the Earth. This helps in the
formation of which type of rocks?

a) Igneous

b) Sedimentary

c) Metamorphic

d) Both (a) and (b)

12
37. Which of the following are the examples of Answer : (c)
Block mountains?

1) Black forest

2) Sierra Nevada

3) Satpura

4) Alpine

Select the correct answer using the codes given


below

a) 1 and 2 b) 2 and 3

c) 1,2 and 3 d) All of these

38. The air that descends on a rain-shadow desert Answer : (c)


is

a) warm and moist

b) cold and dry

c) warm and dry

d) cold and moist

39. All processes that move, elevate or build up Answer : (c)


portions of the earth’s crust come under

a) Orogenic processes

b) Epeirogenic processes

c) Diastrophism

d) Volcanism

13
40. Which one of the following is an example of Answer : (c)
divergent plate boundary?

a) The Andes Mountains of South America

b) The San Andreas fault

c) The Mid-Atlantic Ridge

d) The Himalayas

41. The formation of a river delta involves the Answer : (b)


following processes. In what order do they take
place?

a) Transport, corrasion, deposition

b) Corrasion, transport, deposition

c) Deposition, corrosion, transport

d) Deposition, transport, corrosion

42. Which one among the following rivers is the Answer : (a)
longest?
Nile – Longest
a) Amazon
nd
Amazon – Largest & 2 Longest
b) Amur

c) Congo

d) Lena

14
43. The place where the Blue Nile and White Nile Answer : (c)
effect a confluence is

a) Port Said

b) Cairo

c) Khartoum

d) Alexandria

44. The erosive power of a rivers depends mostly Answer : (d)


upon its

a) width and depth

b) speed and depth

c) gradient and width

d) speed and volume

45. Which one of the following volcanoes is called Answer : (b)


the lighthouse of the Mediterranean?

a) Vesuvius

b) Stramboli

c) Etna

d) Vulcan

15
46. Which of the following is the correct order of Answer : (d)
elements of earth’s crust in decreasing order of
their percentage?
a) Aluminium, iron, silicon, oxygen
b) Oxygen, aluminium, iron, silicon
c) Iron, aluminium, silicon, oxygen
d) Oxygen, silicon, aluminium, iron

47. Match the following list Answer : (c)


List-1 List-2
(Original) (Metamorphic Rock)
A) Basalt 1) Slate
B) Coal 2) Schist
C) Limestone 3) Marble
D) Shale 4) Diamond
Codes
A B C D
a) 1 2 3 4
b) 4 3 2 1
c) 2 4 3 1
d) 3 4 2 1

48. Match the following: Answer : (a)


List-1 List-2
A) Weichert- 1) Lower crust and
Gutenberg upper mantle
B) Mohorovicic 2) Divison of Sial and
(moho) Sima with the crust
C) Lehman 3) Lower mantle and
discontinuity outer core
discontinuity
D) Conrad 4) Outer core and inner
discontinuity core discontinuity

Codes :
A B C D
a) 3 1 4 2
b) 1 2 3 4
c) 2 1 4 3
d) 2 4 1 3

16
49. Which one of the following countries does not Answer : (d)
have a border with China?

a) Myanmar

b) Afghanistan

c) Thailand

d) Kazakhstan

50. Arrange the following in a sequential order Answer : (d)


from bigger to smaller size

1) Sand

2) Granule

3) Cobble

4) Pebble

Select the correct answer using the codes given


below

a) 1,2,3, and 4 b) 4,3,2 and 1

c) 3,4,1 and 2 d) 3,4,2 and 1

51. Consider the following statements Answer : (b)

1) Igneous rocks are rich in natural gas


2) Igneous rocks are rich in metallic minerals
3) Igneous rocks are not fossiferous
4) Igneous rocks consist of silicate minerals
Which of the following statements given above
are correct

a) 1,2 and 3
b) 2,3 and 4
c) 3 and 4
d) All of these

17
52. The Victoria Falls in Africa is located on which Answer : (c)
river?

a) Zaire

b) Orange

c) Zambezi

d) Niger

Answer : (a)

53. What is the technical term for ‘onion peeling’


kind of weathering?

a) Exfoliation

b) Frost Action

c) Solifluction

d) Slumping

54. Which one among the following is the longest Answer : (b)
continuous mountain range in the world?

a) The Himalayas

b) The Andes

c) The Rocky Mountains

d) The Alps

18
55. Which of the following landforms is an Answer : (b)
intermont plateau?

a) The Deccan plateau

b) The Tibetan plateau

c) The Meseta of Spain

d) The Arabian plateau

56. The greatest oceanic depths are found at Answer : (a)

a) Oceanic trenches

b) Abyssal plains

c) A seismic ridges

d) Guyots

57. Which one of the following statements is Answer : (a)


correct with reference to our Solar System

a) The Earth is the densest of all the planets in


our Solar System

b) The predominant element in the composition


of Earth is silicon

c) The Sun contains 75% of the mass of Solar


System

d) The diameter of the sun is 190 times that of


the Earth

19
58. Most of the active volcanoes are found in the Answer : (d)

a) mid-oceanic ridge volcanic zone

b) Sierra Nevada-Cascade volcanic province

c) Mediterranean divergence boundary

d) Circum-Pacific belt

59. Which one of the four regions above Earth has Answer : (d)
smallest height(km)?

a) Stratosphere

b) Mesosphere

c) Thermosphere

d) Troposphere

60. Match the following Answer : (a)

List 1 List 2
A) Ozone hole 1) Ratio of reflected
radiation to radiation
received
B) Greenhouse 2) Acceleration effect of
ozone destruction
C) Global 3) Increase in level
warming
D) Albedo 4) Difference in the
outgoing and incoming
radiation of the
atmosphere

Codes:
A B C D
a) 2 4 3 1
b) 4 3 2 1
c) 3 4 2 1
d) 2 1 3 4

20
61. Which one of the following statements Answer : (c)
regarding ozone layer within the atmosphere
is/are correct?

1) It absorbs most of the ultraviolet radiation


found in the Sun’s rays
2) Chlorofluorocarbons are serious threat to the
ozone layer
Select the correct answer using the codes given below
a) Only 1
b) Only 2
c) Both 1 and 2
d) Neither 1 nor 2

62. What is the reason for the major hot deserts of Answer : (a)
the world lying in the western part of the
tropical latitude?

a) They are influenced by trade winds


b) They lie in the rain-shadow area of the
mountains
c) They are influenced by monsoon winds
d) They are influenced by dry winds

63. Which one of the following statements is true Answer : (b)


about Horse latitude?

a) It is a region of strong wind currents


b) It is a dynamically induced sub-tropical high
pressure
c) It is closest point to the Sun, therefore, the ait
relatively is more hot due to which, the air
becomes less denser and form a low pressure
zone.
d) None of the above

21
64. A layer in the Earth atmosphere called Answer : (b)
lonosphere facilitates radio communication.
Why?
1) The presence of ozone causes the, reflection of
radio waves to Earth
2) Radio waves have a very long wavelength
Which of the above statements given above
is/are correct?
a) Only 1 b) Only 2
c) Both 1 and 2 d) Neither 1 nor 2

65. The jet aircrafts fly very easily and smoothly in Answer : (c)
the lower Stratosphere. What could be the
appropriate explanation?
1) There are no clouds or water vapour in the
lower Stratosphere
2) There are no vertical winds in the lower
Stratosphere
Which one of the statements given above
is/are correct in this context?
a) Only 1
b) Only 2
c) Both 1 and 2
d) Neither 1 nor 2

66. Consider the following statements Answer (c)


1) The difference between the moisture holding
capacity of air and its actual humidity is called
saturation deficit.
2) The temperature to which air has to cooled in
order to reach saturation is called dew point.
Which of the statements given above is/are
correct?
a) Only 1
b) Only 2
c) Both 1 and 2
d) Neither 1 nor 2

22
67. Consider the following statements Answer : (d)
1) A hurricane acquires its spin from the coriolis
effect
2) The diameter of the hurricane decreases as it
moves away from low latitudes.
3) The diameter of a hurricane is never below 150
km
Which of the statements given above is/are correct?
a) Only 1
b) 2 and 3
c) 1 and 3
d) 1, 2 and 3

68. Doldurms is a Answer : (d)


a) tropical wind belt The air at the equator is generally rising for this reason
b) tropical wind deflection belt these is little wind in the region, this region of light
shifting winds near the equator is called the Doldrums.
c) sub-tropical wind belt
d) Tropical no-winded belt

69. Assertion (A) Winds are deflected to their right Answer : (b)
in the Northern Hemisphere and to their left in
the southern Hemisphere
Reason (R) The Earth’s axis is inclined
Codes
a) Both A and R are true, and R is the correct
explanation of A
b) Both A and R are true, but R is not the correct
explanation of A
c) A is true, but R is false
d) A is false, but R is true

23
70. Match the following Answer : (a)
List 1 List 2
(Local Wind) Area of Prevalence
A) Chinook 1) North African desert
B) Foehn 2) Rocky mountain slopes of the
USA
C) Sirocco 3) Northern slopes of Alps
D) Mistral 4) Southern slopes of Alps
A B C D
a) 2 3 1 4
c) 4 1 3 2
b) 2 1 3 4
d) 4 3 1 2

71. What causes wind to deflect toward left in the Answer : (c)
southern hemisphere?
a) Temperature
b) Magnetic field
c) Rotation of the Earth
d) Pressure

72. Which of the following statements characterise Answer : (a)


the anticyclones?
1) Anticyclones are low pressure systems
2) They are characterised by divergent wind
circulation
3) They are indicative of dry weather conditions
Select the correct answer using the codes given
below
a) 2 and 3
b) 1 and 3
c) 1,2 and 3
d) Only 1

24
73. Westerlies in Southern hemisphere are Answer : (a)
stronger and persistent than in Northern
hemisphere. Why?
1) Southern hemisphere as compared to
Northern hemisphere has less landmass.
2) Coriolis force is higher in South hemishere, as
compared to northern hemisphere.
Which of the statement given above is/are
correct?
a) Only 1 b) Only 2
c) Both 1 and 2 d) Neither 1 nor 2

74. Which one among the following statement Answer : (a)


relating to an anticyclone is correct?
a) Anticyclone is a wind system with a high
pressure center
b) In antyicylone the movement of wind is inward
c) The contribution of an anticyclone towards
determining weather of an area is quite
significant
d) The movement of wind is clockwise in an
anticyclone of southern hemisphere

75. Assertion (A) : Pressure gradients determine Answer (c)


the velocity of winds.
Reason (R) : When isobar (lines of equal
atmospheric pressure ) are closely spaced, the
wind velocity would be gentle.
Codes
a) Both A and R are true and R is the correct
explanation of A.
b) Both A and R are true , but R is not the correct
explanation of A.
c) A is true, but R is false
d) A is false but R is true

25
76. Which one among the following is the idealised Answer (c)
global pattern of surface wind from the
Equator to pole?

a) Doldrums-Westerlies-Trade Wind-Easterlies

b) Easterlies- Westerlies-Trade Wind-Easterlies

c) Doldrum-Trade Wind-Westerlies-Easterlies

d) Westerlies-Trade Wind-Doldrum-Easterlies

77. Consider the following statements Answer (d)

1) Biodiversity hotspots are located only in


tropical regions

2) India has four biodiversity hotspots i.e. Eastern


Himalayas, Western Himalayas, Western Ghats
and Andaman and Nicobar Islands

Which of the statements given above is/are


correct?

a) Only 1

b) Only 2

c) Both 1 and 2

d) Neither 1 nor 2

78. What is the name of the winds that blow from Answer (c)
the subtropical high pressure belts towards
the subpolar low pressure belts?

a) Easterlies

b) Trade Winds

c) Westerlies

d) Western disturbances

26
79. Consider the following climatic and Answer (d)
geographical phenomena
1) Condensation
2) High temperature and humidity
3) Orography
4) Vertical Wind
Thunder cloud development is due to which of
these phenomena?
a) 1 and 2
b) 2,3 and 4
c) 1,3 and 4
d) 1,2,3, and 4

80. Match the following Answer (a)


List 1 List 2
(Current) (Feature)
A) Kuroshio current 1) Warm current in the
Atlantic Ocean
B) Peru current 2) Cold current in the
Atlantic Ocean
C) Labrador current 3) Warm current in the
Pacific Ocean
D) Florida current 4) Cold current in the
Atlantic Ocean
Codes
A B C D
a) 3 4 2 1
c) 1 4 2 3
b) 3 2 4 1
d) 1 2 4 3

81. The current is produced by upwelling of cold Answer (b)


water off the coast of Chile and Peru is known
as
a) canary current
b) Humboldt current
c) agulhas current
d) el nino

27
82. Which one of the following pairs of oceans and Answer : (b)
currents is not correctly matched?

a) North Atlantic Ocean : Canary current

b) Eastern Pacific Ocean : Kuroshio current

c) South Atlantic Ocean : Falkland current

d) Indian Ocean : Agulhas current

83. Assertion (A) During the day, winds blow from Answer : (a)
sea to land.

Reason (R) The land gets more heated than


the surrounding sea, hence lower pressure
develops over land as compared to sea.

Codes

a) Both A and R are true, and R is the correct


explanation of A

b) Both A and R are true, but R is not the correct


explanation of A

c) A is true, but R is false

d) A is false, but R is true

84. Which one of following separates North and Answer : (c)


South Islands of Newzealand?

a) Foveaux strait

b) Bass strait

c) Cook strait

d) Torres strait

28
85. Which of the following current is located near Answer : (b)
the coastline of Philippines and China ?

a) Oyashio current

b) Kuroshio current

c) Canary current

d) Benguela current

86. Continental shelf is Answer : (d)

a) a part of oceanic crust made of granite

b) a part of continental crust made of basalt

c) a part of oceanic crust made of basalt

d) a part of continental crust made of granite

87. Which among the following statements about Answer (b)


the North Atlantic Drift is/are correct?

1) It keeps the West coast of northern Europe ice


free

2) It is responsible for the warm air mass which


interacts with the cold air mass from the polar
region and causes rainfall in western Europe/

3) It meets the Labrador current near Vancouver


Island and causes dense fog

Select the correct answer using the codes given below

a) 1,2 and 3 b) 1 and 2

c) Only 2 d) 1 and 3

29
88. Which one of the following countries is not Answer (d)
situated in the region between the Caspian sea
and Black sea?

a) Armenia

b) Azerbaijan

c) Georgia

d) Ukraine

89. The increase in salinity in water Answer : (b)

a) increases the rate of evaporation

b) decreases the rate of evaporation

c) has no effect on the rate of evaporation

d) increases the rate of evaporation initially but


decreases later

90. In which ocean is the Kamchataka current Answer (a)


found?

a) Pacific Ocean

b) Atlantic Ocean

c) Indian Ocean

d) Arctic Ocean

30
91. What is Gulf Stream? Answer (c)

a) A cold current in the Atlantic Ocean

b) A cold current in the Pacific Ocean

c) A warm current in the Atlantic Ocean

d) A warm current in the Pacific Ocean

92. Which one of the following statements is not Answer (a)


correct?

a) Gulf with narrow fronts and wider rears


experience high tides

b) Tidal current takes place when a Gulf is


connected with the open sea by a narrow
channel

c) Tidal bore occurs when a tide enters the


narrow and shallow estuary of a river

d) The tidal nature of the mouth of the river


Hooghly is of crucial importance to Kolkata as
port

93. The Palk Strait lies between Answer : (a)

a) Bay of Bengal and Palk Bay

b) Andaman and Nicobar Islands

c) Rann of Kutchchh and Gulf of Khambhat

d) Lakshadweep and Maldives

31
94. Consider the following countries Answer : (d)

1) Australia

2) Namibia

3) Brazil

4) Chile

Through which of the above does the Tropic of


Capricorn pass

a) Only 1 b) 2, 3 and 4

c) 1, 2 and 3 d) All of these

95. Through which one of the following continents, Answer : (a)


do the equator, the Tropic of Cancer and The
Tropic of Capricorn pass through?

a) Africa

b) South America

c) North America

d) Australia

96. Which one of the following countries is Answer (b)


credited with the establishment of ‘Uranium
city’?

a) Australia

b) Canada

c) Russia

d) USA

32
97. Which of the following countries does not border Answer : (d)
Brazil?

a) Argentina

b) Bolivia

c) Uruguay

d) Ecuador

98. Which one of the following countries is land Answer : (a)


locked?

a) Bolivia

b) Peru

c) Surinam

d) Uruguay

99. The Narmada river flows to the west, while Answer : (a)
most other large peninsular rivers flow to the
East. Why?

1) It occupies a linear Rift Valley

2) It flows between the Vindhyas and the


Satpuras

3) The land slopes to the West from Central India

Select the correct answer using the codes given below

a) Only 1 b) 2 and 3

c) 1 and 3 d) None of these

33
100. The latitudes that pass through Sikkim also Answer : (a)
pass through

a) Rajasthan

b) Jammu and Kashmir

c) Himachal Pradesh

d) Punjab

101. Consider the following statements: Answer : (b)

1. Igneous rocks have strata like sedimentary


rocks.

2. Igneous rocks do not contain fossils.

Which of the statements given above is/are


correct?

(a) 1 only

(b) 2 only

(c) Both 1 and 2

(d) Neither 1 nor 2

102. Consider the following pairs: Answer : (a)

1. Simpson Desert - China


2. Atacama Desert - Chile and Peru
3. Takla Makan - Australia
Which of the pairs given above is/are correctly
matched?

(a) 2 only
(b) 1 and 2
(c) 2 and 3
(d) 1, 2 and 3

34
103. Consider the following statements: Answer : (c)
1. ‘Batholiths’ are long, irregular and undulating
forms of solidified intruded magmas.
2. ‘Dykes’ represent wall-like formation of
solidified magmas.
3. Thick beds of magmas are called ‘Sheets’.
Which of the statements given above is/are correct?
(a) 1 only (b) 2 only
(c) 1 and 2 (d) 1, 2 and 3

104. What is meant by the eccentricity of the orbit Answer : (d)


of the earth?
(a) Change in the distance between the earth and
the sun due to the elliptical shape of the orbit
of the earth.
(b) Reversal of the dates of aphelion and
perihelion.
(c) Changes in the angle of incidence of the
earth's axis.
(d) Change in the shape of the orbit of the earth.

105. Which of the following is/are the native tribes Answer : (d)
of North America?
(1) Apache
(2) Blackfoot
(3) Cheyenne
Select the correct answer using the codes given below:
(a) 1 only (b) 1 and 2 only
(c) 2 and 3 only (d) All of the above

35
106. Consider the following pairs: Answer : (a)
1. Mt. Etna Volcano - Italy
2. Mt. Kilimanjaro Volcano - South Africa
3. Stromboli Volcano - Indonesia
Which of the pairs given above is/are correctly
matched?
(a) 1 only (b) 2 only
(c) 1 and 3 (d) 1, 2 and 3

107. Consider the following statements with respect Answer : (b)


to Turkey:
1. The Black Sea and the Caspian Sea touch the
boundaries of Turkey.
2. The Dardanelles straitis a significant waterway
in northwestern Turkey that forms part of the
continental boundary between Europe and
Asia.
Which of the statements given above is/ are correct?
(a) 1 only (b) 2 only
(c) Both 1 and 2 (d) Neither 1 nor 2

108. Which of the following conditions is/are Answer : (b)


needed for the formation of the tropical
cyclones ?
1. Presence of warm oceanic surface with a
o
temperature of 27 C or more.
2. Coriolis force.
3. Pre-existing high pressure area.
Select the correct answer using the code given below:
(a) 1 only
(b) 1 and 2
(c) 3 only
(d) 1, 2 and 3

36
109. In which of the following seas, Cyprus Island is Answer : (c)
located?

(a) Caspian Sea

(b) Black Sea

(c) Mediterranean Sea

(d) Aral Sea

110. In which of the following countries Hairatan is Answer : (c)


located (which was recently in news due to
One Belt One Road initiative)?

(a) Kyrgyzstan

(b) China

(c) Afghanistan

(d) Uzbekistan

111. Which one pair of the following places is linked Answer : (c)
by Channel Tunnel?

(a) London - Berlin

(b) Berlin - Paris

(c) Kent - Pas-de-Calais

(d) Barcelona - Berlin

37
112. With reference to air mass, which of the Answer : (c)
following statements is/are correct?

1. An air mass is a large volume of air in the


atmosphere that is mostly uniform in
temperature and moisture.

2. Air masses are separated from each other by


boundaries called ‘fronts’.

Select the correct answer using the code given below:

(a) 1 only (b) 2 only

(c) Both 1 and 2 (d) Neither 1 nor 2

113. Folds in the mountains are the results of: Answer : (a)

(a) Compression Force

(b) Parallel Force

(c) Shear Force

(d) Tensional Force

114. Which one of the following is the dominant Answer : (c)


element of the Earth crust?

(a) Aluminium

(b) Iron

(c) Oxygen

(d) Silicon

38
115. Match List I with List II and select the correct Answer : (c)
answer using the code given below the lists:

List I List-II
(Original Rock) (Metamorphic
Rock)

A. Basalt 1. Diamond

B. Coal 2. Marble

C. Limestone 3. Slate

D. Shale 4. Schist

Code:
A B C D
(a) 3 4 2 1
(c) 4 1 2 3
(b) 3 2 4 1
(d) 4 1 3 2

116. The term “outwash plain” is associated with Answer : (a)


(a) Glaciers
(b) Rivers
(c) Sea Waves
(d) Wind Action

117. The Moho Discontinuity separates Answer : (b)


(a) Asthenosphere and mantle
(b) Mantle and lower crust
(c) Mantle and core
(d) Inner solid core and outer liquid core

39
118. The term “adiabatic change of temperature” Answer : (d)
stands for

(a) Heating of ascending air

(b) Heating of descending air

(c) Cooling of ascending air

(d) Heating or cooling of ascending or descending


air through expansion or compression

119. The widest continental shelf is found in the Answer : (a)

(a) Arctic Ocean

(b) Atlantic Ocean

(c) Indian Ocean

(d) Pacific Ocean

120. With reference to the significance of the ‘jet Answer : (c)


streams’, which of the following statements
is/are correct?

1. There is a close relationship between the


intensity of the temperate cyclones and the jet
streams.

2. The monsoon of South Asia is largely affected


and controlled by the jet streams.

Select the correct answer using the code given below:

(a) 1 only (b) 2 only

(c) Both 1 and 2 (d) Neither 1 nor 2

40
121. Which one of the following is a cold water Answer : (c)
current?

(a) Agulhas

(b) Guinea

(c) Humboldt

(d) Kuro-siwo

122. Which one of the following countries is the Answer : (c)


largest producer of Uranium?

(a) Australia

(b) Canada

(c) Kazakhstan

(d) Niger

123. The equator does not pass through which of Answer : (c)
the following countries?

(a) Maldives

(b) Indonesia

(c) Malaysia

(d) Somalia

41
124. Which of the following lakes lies on the Answer : (a)
equator?
(a) Lake Victoria
(b) Lake Malavi
(c) Lake Nasser
(d) None of these

125. Which of the following statements about Answer : (a)


salinity is not correct?
(a) Salinity is directly related to precipitation.
(b) There exists a direct relationship between the
rate of evaporation and salinity.
(c) Low salinity is found near the mouth of a river.
(d) The salinity is the highest at the tropics and
decreases towards the poles and the equator.

126. Match List I with List II and select the correct Answer : (c)
answer using the code given below the lists:

List I (Sea) List II (Country)

A. Black Sea 1. Bulgaria

B. Red Sea 2. China

C. Yellow Sea 3. Saudi Arabia

D. Caspian Sea 4. Kazakhstan

Code:
A B C D
(a) 1 4 2 3
(b) 2 3 1 4
(c) 1 3 2 4
(d) 2 4 1 3

42
127. Consider the following pairs: Answer : (c)
Regions sometimes Country
mentioned in the news
Catalonia : Spain
Crimea : Hungary

Mindanao : Philippines

Oromia : Nigeria

Which of the pairs given above are correctly


matched?
(a) 1, 2 and 3 (b) 3 and 4
(c) 1 and 3 (d) 2 and 4

128. Which one of the following pairs is not Answer : (c)


correctly matched?
(a) Bahamas : Nassau
(b) Costa Rica : San Jose
(c) Nicaragua : Belmopan
(d) Dominican Republic : Santo Domingo

129. Match List – I (City) with List – II (River) and Answer : (d)
select the correct answer using code given
below the lists:
List I (City) List II (River)

A. Washington D. C. 1. River Manzanares

B. Berlin 2. River Seine

C. Paris 3. River Spree


D. Madrid 4. River Potomac

Code:
A B C D
(a) 2 3 4 1
(c) 2 1 4 3
(b) 4 1 2 3
(d) 4 3 2 1

43
130. Consider the following statements: Answer : (a)
1. The earth’s magnetic pole in the northern
hemisphere is located on a peninsula in
northern Canada.
2. Earth’s magnetic equator passes through
Bhopal in India.
Which of the statements given above is/are
correct?
(a) 1 only
(b) 2 only
(c) Both 1 and 2
(d) Neither 1 nor 2

131. Which one of the following is the correct Answer : (b)


sequence of the given continents in the
decreasing order of their percentage of Earth’s
land?
(a) North America – Africa – South America –
Europe
(b) Africa – North America – South America –
Europe
(c) North America – Africa – Europe– South
America
(d) Africa – North America – Europe– South
America

133. The most important fishing grounds of the Answer : (c)


world are found in the regions where
(a) warm and cold atmospheric currents meet.
(b) rivers drain out large amounts of fresh water
into the sea.
(c) warm and cold oceanic currents meet.
(d) continental shelf is undulating

44
134 Which of the following is/are unique Answer : (d)
characteristic/ characteristics of equatorial
forests?
1. Presence of tall, closely set trees with crowns
forming a continuous canopy.
2. Coexistence of a large number of species.
3. Presence of numerous varieties of epiphytes.
Select the correct answer using the code given
below:
(a) 1 only
(b) 2 and 3
(c) 1 and 3
(d) 1, 2 and 3

135. "Climate is extreme, rainfall is scanty and the Answer : (d)


people are used to be nomadic herders."
The above statement best describes which of
the following regions?
(a) African Savannah
(b) Central Asian Steppe
(c) North American Prairie
(d) Siberian Tundra

136. During a thunderstorm, the thunder in the Answer : (c)


skies is produced by the
1. meeting of cumulonimbus clouds in the sky.
2. lightning that separates the nimbus clouds.
3. violent upward movement of air and water
particles.
Select the correct answer using the code given
below :
(a) 1 only
(b) 2 and 3
(c) 1 and 3
(d) None of the above produces the thunder

45
137. Consider the following statements: Answer : (d)
1. The Arabian Sea side witnesses more cyclones
than the Bay of Bengal side, on average.
2. The cyclone ‘Ockhi’ originated near the western
coast of Odisha.
Which of the statements given above is/are
correct?
(a) 1 only
(b) 2 only
(c) Both 1 and 2
(d) Neither 1 nor 2

138. Which among the following straits connects Answer : (a)


the Mediterranean Sea with the Atlantic
Ocean?
(a) Strait of Gibraltar
(b) Strait of Magellan
(c) Strait of Panama
(d) None of the above

139. What are varves? Answer : (b)


(a) Annual change in temperature from summer
to winter.
(b) Annual layers of sediment in lakes
experiencing annual freezing and thawing.
(c) Layers of sediment contained in deep ice cores
of the Arctic region.
(d) Sequence of fossils in a chronological order.

46
140. The phenomenon of El-Nino is associated with Answer : (c)
the

(a) Agulhas Current

(b) Benguela Current

(c) Humboldt Current

(d) East Australian Current

141. Which of the following mountains is/are Fold Answer : (d)


Mountains?

1. Andes

2. Alps

3. Himalayas

Select the correct answer using the code given below:

(a) 1 only

(b) 2 only

(c) 3 only

(d) 1, 2 and 3

142. Which of the following industries is most Answer : (b)


developed in the Great Lakes region of North
America?

(a) Textile and chemicals

(b) Steel and engineering

(c) Cement and paper

(d) Film industry

47
143. The conservation of Galapagos Islands was Answer : (d)
in news recently. Consider the following
statements with respect to Galapagos
Islands

(1) Galapagos Islands are located off coast of


Ecuador.

(2) It is a UNESCO world heritage site.

(3) They are of volcanic origin.

Which of the statements given above is/are


correct?

(a) 1 and 2 (b) 2 and 3

(c) 1 and 3 (d) 1, 2 and 3

144. Which of the following landform is not Answer : (b)


associated with glaciations?

(a) Drumlin

(b) Inselberg

(c) Moraines

(d) Hanging valley

145. Tides occur in the oceans and the seas due to Answer : (d)
which among the following?

1. Gravitational force of the Sun.

2. Gravitational force of the Moon.

3. Gravitational force of the Earth.

Select the correct answer using the code given below:

(a) 1 only (b) 2 and 3

(c) 1 and 3 (d) 1, 2 and 3

48
146. The proposed South Asia Gas Enterprise Answer : (b)
(SAGE) pipeline will pass through which of the
following countries?
1. India
2. Oman
3. Iran
4. Afghanistan
Select the correct answer using the code given below :
(a) 1 and 2 (b) 1, 2 and 3
(c) 2, 3 and 4 (d) 1, 2, 3 and 4

147. In the South Atlantic and the South Eastern Answer : (a)
Pacific regions in the tropical latitudes,
cyclones do not originate. What is the reason?
(a) Sea surface temperatures are low.
(b) The ‘Inter Tropical Convergence Zone’ (ITCZ)
seldom occurs.
(c) Coriolis force is too weak.
(d) Absence of land in these regions.

148. Consider the following statements about coral Answer : (c)


reefs:
1. Coral bleaching occurs when corals lose their
zooxanthellae, exposing the white calcium
carbonate skeletons of the coral colony.
2. Corals are sessile animals, meaning they are
not mobile, but stay fixed in one place.
Which of the statements given above is/are
correct?
(a) 1 only (b) 2 only
(c) Both 1 and 2 (d) Neither 1 nor 2

49
149. The boundary of Israel touches the boundaries Answer : (c)
of which of the following countries?
1. Lebanon
2. Egypt
3. Jordan
4. Saudi Arabia
Select the correct answer using the code given
below:
(a) 1 and 4
(b) 2 and 3
(c) 1, 2 and 3
(d) 1, 2, 3 and 4

150. The Genetic Engineering Appraisal Committee Answer : (c)


is constituted under the
(a) Food Safety and Standards Act, 2006
(b) Geographical Indications of Goods
(Registration and Protection) Act, 1999
(c) Environment (Protection) Act, 1986
(d) Wildlife (Protection) Act, 1972

151. How does National Biodiversity Authority (NBA) Answer : (c)


help in protecting the Indian agriculture?
1. NBA checks the biopiracy and protects the
indigenous and traditional genetic resources.
2. NBA directly monitors and supervises the
scientific research on genetic modification of
crop plants.
3. Application for intellectual Property Rights
related to genetic / biological resources cannot
be made without the approval of NBA.
Which of the statements given above is/are correct?
(a) 1 only
(b) 2 and 3 only
(c) 1 and 3 only
(d) 1, 2 and 3

50
152. The National Green Tribunal Act, 2010 was Answer (a)
enacted in consonance with which of the
following provisions of the Constitution of
India?
1. Right to healthy environment, construed as a
part of part of Right to life under Article 21.
2. Provision of grants for raising the level of
administration in the Scheduled Areas for the
welfare of Scheduled Tribes under Article
275(1)
Which of the statements given above is/are correct?
(a) 1 only
(b) 2 only
(c) Both 1 and 2
(d) Neither 1 nor 2

153. Consider the following statements: Answer : (b)


(1) Animal Welfare Board of India is established
under the Environment (Protection) Act, 1986.
(2) National Tiger Conservation Authority is a
statutory body.
(3) National Ganga River Basin Authority is chaired
by the Prime Minister.
Which of the statements given above is/ are correct?
(a) 1 only
(b) 2 and 3 only
(c) 2 only
(d) 1, 2 and 3

154. With reference to Bombay Natural History Answer : (c)


Society (BNHS), consider the following
statements:
(1) It is an autonomous organization under the
Ministry of Environment and Forests.
(2) It strives to conserve nature through action-
based research, education and public
awareness.
(3) It organizes and conducts nature trails and
camps for the general public.
Which of the statements given above is/are correct?
(a) 1 and 3 only
(b) 2 only
(c) 2 and 3 only

51
(d) 1, 2 and 3
155. Which of the following can be found as Answer : (c)
pollutants in the drinking water in some
parts of India?
1. Arsenic 2. Sorbitol
3. Fluoride 4. Formaldehyde
5. Uranium
Select the correct answer using the codes given below.
(a) 1 and 3 only (b) 2, 4 and 5 only
(c) 1, 3 and 5 only (d) 1, 2, 3, 4 and 5

156. Recently, ‘oilzapper’ was in the news. What Answer : (a)


is it?
(a) It is an eco-friendly technology for the
remediation of oil sludge and oil spills.
(b) It is the latest technology developed for under-
sea oil exploration.
(c) It is a genetically engineered high biofuel
yielding maize variety.
(d) It is the latest technology to control the
accidentally caused flames from oil wells.

157. Acid rain is caused by the pollution of Answer : (d)


environment by Carbon dioxide, carbon monoxide, oxides of
(a) carbon dioxide and nitrogen nitrogen and oxides of sulphur in atmosphere can
(b) carbon monoxide and carbon dioxide cause acid rains. Both b) and d) are correct. But d)
has more influence than b) Answer: d) nitrous
(c) ozone and carbon dioxide
oxide and sulphur dioxide
(d) nitrous oxide and sulphur dioxide
Types of Acid Deposition

 “Acid rain” is a broad term referring to a mixture


of wet and dry deposition (form of deposition
material) from the atmosphere.
Wet Deposition

 If the acid chemicals in the air are blown into


areas where the weather is wet, the acids can fall
to the ground in the form of rain, snow, fog, or
mist.
 As this acidic water flows over and through the
52
ground, it affects a variety of plants and animals.
Dry Deposition

 In areas where the weather is dry, the acid


chemicals may become incorporated into dust or
smoke and fall to the ground through dry
deposition, sticking to the ground, buildings,
vegetation, cars, etc.
 Dry deposited gases and particles can be washed
from these surfaces by rainstorms, through
runoff. This runoff water makes the resulting
mixture more acidic.
 About half of the acidity in the atmosphere falls
back to earth through dry deposition.

Q158. The acidification of oceans is increasing. Answer (d)


Why is this phenomenon a cause of
concern?
1. The growth and survival of calcareous
phytoplankton will be adversely affected.
2. The growth and survival of coral reefs will be
adversely affected.
3. The survival of some animals that have
phytoplanktonic larvae will be adversely
affected.
4. The cloud seeding and formation of clouds will
be adversely affected.
Which of statements given above is / are correct ?
(a) 1, 2 and 3 only (b) 2 only
(c) 1 and 3 only (d) 1, 2, 3 and 4

159. Suppose you are travelling along the Answer : (d)


Eastern Ghat on Eastern Coast of India from
Northern to Southern region, the sequence
in which you will cross these rivers
(a) Krishna-Godavari-Mahanadi-Kaveri
(b) Mahanadi-Krishna -Godavari-Kaveri
(c) Krishna-Godavari-Mahanadi-Kaveri
(d) Mahanadi-Godavari-Krishna-Kaveri

53
160. Consider the statements with respect to Answer : (d)
Ganges Drainage System and Peninsular Explanation:
Drainage System
Damodar River, a river Ganges River System occupies
1. Peninsular rivers bear the exception of flowing
the eastern margins of the Chotanagpur Plateau
through Rift valleys, like Narmada, while
where it flows through a rift valley. So, first statement
Ganges Rivers have no such feature.
is wrong. Peninsular rivers are characterized by fixed
2. All Peninsular rivers are characterized by fixed
course, absence of meanders and non-perennial flow
course, absence of meanders and non-
of water. The Narmada and Tapi which flow through
perennial flow of water while Ganges rivers are
the rift valley are, however, exceptions. Hence none of
perennial, having meanders and have non-
the statements are correct.
fixed courses.
Select the correct statement/s
(a) Only 1 (b) Only 2
(c) Both (d) None
161. Heat waves was in news recently which killed Answer : (a)
more than 2,500 people till June. Consider the Heat waves impacted mainly in northern states like
following statements regarding heat waves Uttar Pradesh, Bihar and Eastern states like Odisha,
1) Heat wave's adverse impact occurred mainly in Andhra Pradesh, Telangana.
Northern and Eastern region of India. Central government has declared cold waves, tsunamis
2) Severe heat waves come under National etc as National Calamity but not heat waves.
Calamity just like cold waves. Thus recently after the heat waves took toll of many
3) IMD recommended to declare a region as severe lives country wide, there was a demand from experts
heat waves affected region if temperature as well as many states for heat waves to be declared as
reached beyond 45 degree Celsius. National Calamity.
4) It is a natural calamity found in tropical regions Heat waves are the phenomenon found not only in
and not in temperate regions. tropical regions but also in temperate regions.
Choose the correct statement/s from the code Example: UK experienced the severe heat waves in
(a) 1, 3 only (b) 2, 3, 4 only 2013.
(c) 1, 2, 3 only (d) All the above

162. Consider the following statements regarding Answer : (c)


tropical cyclone which is one of the The energy is derived from the condensation of water
important determining factors for the vapor into water droplets and of this only meager 3% is
success of Indian monsoons. utilized for sustaining the wind movements.
1) These are termed as the heat engines, which
derive their energy from the condensation of
water vapour into water droplets/drops.
2) More than 50% of the energy derived is utilized
to sustain the wind movement around the
central eye.
3) A mature cyclone releases energy equivalent to
that of 100 hydrogen bombs.
Which of the following statements are NOT true
(a) 1, 2 only (b) 1, 3only
(c) 2 only (d) None of the above

54
163. Consider the following statements Answer : (a)
regarding biogas which is a very important Biogas is a type of gas that is formed by the biological
source of energy in rural areas. breakdown of organic matter in an oxygen deficient
1. Biogas has a higher thermal efficiency environment. It is counted as an eco friendly bio fuel.
compared to charcoal, cow dung, kerosene. Biogas contains 60% methane and carbon dioxide. It
2. Not only biogas serves as a source of energy in can be employed for generating electricity and also as
the form of fuel but also can be used as source automotive fuel. For example; each household builds
of manure for farm crops. its own plant to channel waste from the domestic toilet
3. It is slightly less environment friendly as it and nearby shelters for animals, usually pigs, cows into
releases green house gases when burnt as a a sealed tank.
fuel. The waste ferments and is naturally converted into gas
Choose the incorrect code from the following and compost, resulting in improved sanitary conditions
(a) 1, 2 only (b) 2, 3 only at home.
(c) 3 only (d) None of the above A large amount of straw, which was previously burned,
is now put into biogas tanks to ferment. This reduces
air pollution from smoke and helps produce high-
quality organic fertilizer.
Biogas plants significantly lower the greenhouse
effects on the earth's atmosphere. The plants lower
methane emissions by entrapping the harmful gas and
using it as fuel. Thus it is an environment friendly fuel.
164. Why black soils are treated with cement or Answer : (d)
hydrated lime during Roadways and The cement or lime treatment is being utilized for the
railways construction? Consider the following purposes:
reasons 1. To provide a pavement foundation of marginally
1. To overcome problems of volume changes due weaker in strength than that of concrete pavement, but
to alternate expansion and shrinkage of soils. much improved strength than natural Black cotton soil
2. To prevent soil erosion during wet season. (BC soil).
3. To improve bearing capacity of the soil. 2. To consolidate subgrades and base courses for
Choose the correct reason from the codes below. concrete pavement in order to make them resistant to
(a) 1, 2 only (b) 2, 3 only volume changes and displacement or erosion in the
(c) 1, 3 only (d) All the above presence of moisture even under the rocking action of
curled slabs, if any.
3. To overcome the susceptibility of foundations to
volume change
4. and to increase their shearing resistance and bearing
capacity.

55
165. Left hand side is the region and right hand Answer : (a)
side is the major reason in that region for Overgrazing is mainly the reason in the western India
soil degradation.
i.e. Rajasthan, Maharastra while over irrigation is the
1. Odisha, Jharkhand — deforestation and major reason in the North Western India ie Punjab,
mining. Haryana.
2. Rajasthan, Maharashtra — over irrigation.
3. Punjab, Haryana — overgrazing.
Which are correctly matched?
(a) 1 only
(b) 1, 2 only
(c) All the above
(d) None of the above

166. Black soils are known for its fertility with Answer : (a)
little or no evidence of exhaustion. This is
Black soil is mainly composed of clay, thus it has
because
high moisture retention capacity. Black soils are in
1. Black soils with equal proportion of clay, gravel
fact rich in kankar nodules which add to the fertility
and coarse sand has high moisture retention
capacity.
of the soil.
2. Cracks developed during dry season allow
oxygenation of the soils.
3. Black soil has low level of kankar thus supports
wide variety of crops.
Choose the incorrect reason from the codes below
(a) 2 Only (b) 2, 3 only
(c) 1, 3 only (d) All the above

56
167. Consider the statements regarding Answer : (d)
salinization which is a one of the major In drier regions wherever salt accumulation has
problem, reducing Indian agricultural occurred because of capillary action, the salts are
productivity. brought from the shallow depth where the saline
1. In drier regions, capillary action can lead to salt ground water table exists.
accumulation on the surface of the land which is Salination can be caused by natural processes such
brought from the deep saline ground water as mineral weathering or by the gradual withdrawal
tables. of an ocean. It can also come about though artificial
2. Irrigation can result in salinization of soils. processes such as irrigation.
3. It can occur both naturally as well as artificially. Salinity from irrigation can occur over time wherever
Choose the correct statements from the codes given irrigation is used; since almost all water (even natural
below. rainfall) contains some dissolved salts. When the
(a) 1, 2 only (b) 1, 3 only plants use the water, the salts are left behind in the
(c) 2, 3only (d) All the above soil and eventually begin to accumulate.

168. In India El Nino can have comparatively Answer : (b)


lesser impact on Sugarcane and Urad More than 90% of the area under sugarcane is under
production. irrigation and thus rainfall failure cannot much
What are the reasons? impact production provided electricity and water
1) Sugarcane is largely grown under irrigated areas. supply is made available for the farmers.
2) Urad is mainly grown in regions where rainfall Urad is drought resistant crop thus grown in regions
variability is very low. even if the rainfall variability is very high.
3) Urad is drought resistant crop.
4) Sugarcane although a khariff crop requires very
less water.
Choose the right pair of reasons from the codes
below
(a) 1, 2 (b) 1, 3
(c) 2, 4 (d) 3, 4

169. Indian agriculture is termed to be an Answer : (b)


Intensive Subsistence Farming type. What are The following are the characteristics of the Intensive
the characteristic that has made Indian Subsistence Farming.
agriculture to be categorized into that type? Land holdings are very small due to inheritance law
1) Very high population density. prevalent in the country and high population mounts
2) Labour intensive farming. pressure on the land holdings to be utilized in an
3) Lower yield of production in terms of per capita intensive way so as to cater the demand for food
labour force. from huge population. Farmers work with the help of
4) Very small and fragmented land holdings. family labour and use of machinery is limited and
5) Usage of farm machinery. most of the agricultural operations are done by
Choose the correct code from the following. manual labour. Farm yard manure is used to
(a) 1, 2, 3 only (b) 1, 2, 3, 4 only maintain the fertility of the soil. In this type of
(c) 2, 3, 4 only (d) All the above agriculture, the yield per unit area is high but per
labour productivity is low.

57
170. What are the factors that have favoured Answer : (a)
the Wheat production in North West India? North West India does have semi arid region but that
1) Western disturbances. itself is not a favourable characteristic, it’s the Western
2) Fertile alluvial soil brought down mainly by disturbance that favours Wheat production.
Indus and Ganga river system.
3) Semi arid region.
Choose the correct code from the following.
(a) 1, 2 only
(b) 2, 3 only
(c) 1, 3 only
(d) All the above

171. Consider the statements regarding a Answer : (a)


particular valley which was in the news
recently.
1) It is a part of Gangotri National Park.
2) It is a cold desert and looks like a replica of
Tibetan plateau.
3) It is home for Snow Leopard.
Which is that valley?
(a) Nellong valley
(b) Kashmir valley
(c) Alaknanda valley
(d) Bhagirathi valley

172. In Tamil Nadu region, evergreen forests are Answer : (d)


found despite hot and dry summers with Humidity is the main reason for trees not shedding
temperature of about 28 degree Celsius.
their leaves during prolonged dry seasons. Red and
What is/are the reason/s? Yellow soils are not good at retaining moisture unlike
(a) Annual rainfall of about 100cms mostly from Black soil which is very good at retaining moisture.
the North East monsoon winds during October
to December.
(b) A humidity of more than 75% throughout the
year.
(c) Red and yellow soil found in Tamil Nadu region
can retain moisture well.
(d) Both 1 and 2

58
173. Migration is an unavoidable worldwide Answer : (b)
phenomenon. Human migration is the movement by people from
Which among the following is/are the not one place to another with the intention of settling
the push factors for rural to urban temporarily or permanently in the new location. The
migration especially in India. movement is typically over long distances and from
a) Unemployment. one country to another, but internal migration is also
possible.
b) Health and education facilities.
Migration may be individuals, family units or in large
c) Monsoon vagaries. groups.
d) Caste disabilities. There are 2 factors ie push and pull factor which might
be real or perceptional. Push factors are those that
force the individual to move voluntarily, and in many
cases, they are forced because the individual risk
something if they stay.
Pull factors are those factors in the destination region
that attract the individual or group to leave their
home. Those factors are known as place utility, which
is the desirability of a place that attracts people. Better
economic opportunities, more jobs, and the promise
of a better life like better health and education
facilities often pull people into new locations.

174. Consider the statements regarding our Answer : (d)


Solar System. Pluto is the second largest dwarf planet next only to
1) Pluto is the largest dwarf planet revolving Eris. Venus and Uranus’ rotatory motion is different
around the Sun. from rest of the planets i.e. from East to West.
2) Every planet except Venus rotates from West Hydrogen nuclei combine to form helium inside the
to East just like Earth on its own axis. Sun under high temperature and pressure and
releases huge heat energy and this process is nuclear
3) It is nuclear fusion which takes place within the
fusion.
Sun which releases huge heat energy.
Choose the correct statements from the following
code.
a) 1, 2 only b) 1, 3 only
c) 2, 3 only d) 3 only

175. Earth although has an average density of Answer : (a)


3
5.5 g/cm , its core has a density of more High temperature need not result into higher density.
3
than 11 g/cm .
What are the possible reasons?
1) Core is formed of materials like iron, nickel etc
which has higher density.
2) Incumbent pressure from the overlaying rocks.
3) High temperature prevalent in the core region

59
[more than 5000 degree Celsius].
Choose the correct code from the following.
a) 1, 2 only b) 2, 3 only
c) 1, 3 only d) All the above

60
176. Earth experiences seasons in a rhythmic Answer : (d)
fashion. What are the causative factors? Axial parallelism is the unchanging orientation of the
1) Rotation of the Earth on its own axis. Earth’s axis. Earth’s North Pole is currently pointing to
2) Revolution of the Earth in its orbit around the the Polar star.
Sun. All the above reasons combine and result a into
3) Spherical shape of the Earth. season formation.
4) Earth's tilted axis.
5) Axial parallelism.
Choose the correct code from the following.
a) 1, 2 only b) 1, 2, 3 only
c) 1, 2, 4, 5 only d) All the above

177. Earth's magnetosphere is the area around Answer : (b)


the Earth where the geomagnetic field First statement is wrong because the magnetosphere
stretches out into space. Consider the stretches away from the Sun and not towards the Sun.
following statements regarding The solar wind is a stream of plasma released from the
magnetosphere. upper atmosphere of the Sun. It consists of mostly
1) It is uneven in shape as it stretches largely electrons, protons and alpha particles with energies
towards the Sun because of Sun's magnetic field. usually between 1.5 and 10 keV.
2) Solar winds can have influence on the Earth's As the solar wind approaches a planet that has a well-
magnetosphere. developed magnetic field (such as Earth, Jupiter and
3) This sphere deflects the charged particles that Saturn), the particles are deflected by the Lorentz
approach the Earth's surface. force. This region, known as the magnetosphere,
Choose the correct statement/s from the following causes the particles to travel around the planet rather
codes. than bombarding the atmosphere or surface. The
a) 1, 2 only b) 2, 3 only magnetosphere is roughly shaped like a hemisphere
c) 1, 3 only d) All the above on the side facing the Sun, then is drawn out in a long
wake on the opposite side. The boundary of this
region is called the magnetopause, and some of the
particles are
able to penetrate the magnetosphere through this
region by partial reconnection of the magnetic field
lines.

178. Sun spots are the one found on the outer Answer : (b)
surface of the Sun. Consider the statements Sunspots are temporary phenomena on the
regarding Sun spots. photosphere of the Sun that appear visibly as dark
1) Sun spots region generally have lower spots compared to surrounding regions. They
temperature than the surrounding region on the correspond to concentrations of magnetic field that
Sun's outer surface. inhibit convection and result in reduced surface
2) Sun spots do not have any effect on Earth's temperature compared to the surrounding
climate. photosphere. Sunspots usually appear as pairs, with
3) Each period of weak Sun spot activity co-relates each spot having the opposite magnetic polarity of the
with the period of overall high solar radiation other.
output from the Sun's surface. Sunspot populations quickly rise and more slowly fall
4) Sun spots generally have 11 year cycle. on an irregular cycle of 11 years, although significant
61
Choose the correct statement/s from the following variations in the number of sunspots attending the 11-
codes. year period are known over longer spans of time.
a) 1, 2 only b) 1, 3, 4 only Since sunspots are darker than the surrounding
c) 1, 2, 4 only d) 2, 4 only photosphere it might be expected that more sunspots
would lead to less solar radiation and a decreased
solar constant. However, the surrounding margins of
sunspots are brighter than the average, and so are
hotter; overall, more sunspots increase the Sun's solar
constant or brightness.
Thus sun spots do have effect on the Earth’s climate
because it varies the solar radiation output thus
affecting the insolation reaching on the Earth’s surface.

62
179. Consider the statements regarding geysers which Answer : (d)
are special type of hot springs. A geyser is a spring characterized by
1) Hot water and vapour spouts from geyser tube intermittent discharge of water ejected
intermittently. turbulently and accompanied by steam.
2) It represents the minor form of broader processes of Over one thousand known geysers exist
vulcanicity. worldwide. At least 1,283 geysers have erupted
3) It is found only in tropical regions. in Yellowstone National Park, Wyoming, United
Choose the incorrect statement/s from the following States, Valley of geysers, Russia etc. Thus it is
code. found also in the temperate regions.
a) 2, 3 only
b) 1, 2 only
c) 2 only
d) 3 only

180. Consider the statements regarding volcanoes. Answer : (d)


1) More than 95% of the global volcanoes are along the In the convergent plate boundaries, one plate is
plate boundaries. subdued by the other plate and the plate which
2) High intensity volcano types like Visuvius type, is pushed underneath the other plate goes to
Peelean type etc are concentrated in the convergent greater depth and starts melting, this result in
plate boundaries. high intensity volcanoes.
3) Low intensity volcano like Mid Atlantic Ridge are While in divergent plate boundaries, the plate
concentrated in the divergent plate boundary move away from each other thus thinning the
regions. crust and thus volcano rises easily to the
Choose the correct statement/s from the following surface without any force being needed to apply
codes. to rise to the Earth’s surface.
a) 1, 2 only b) 2, 3 only
c) 1, 3 only d) All the above

181. Mass wasting or mass movement is a common Answer : (a)


phenomenon at the foothills of big, steep Rock debris move under the influence of
mountains like Himalayas, Atlas etc. Gravitational force and this movement is aided
Consider the following statements regarding or triggered by volcano, earthquake, moving
mass wasting. water etc.
1) Rock debris that moves down the hill is mainly
derived from weathering.
2) The rock debris moves down mainly because of
earthquake, volcano, moving water etc.
Choose the correct statement/s from the following
codes.
a) 1 only b) 2 only
c) Both d) None of the above

63
182. Arcuate deltas are the most common type of Answer : (d)
deltas, example being Ganga delta, Rhine delta, It is known as growing delta because the
Nile delta etc.
sedimentation from the river grows towards the
Consider the following statements regarding sea every year. Delta formed when river water is
this type of delta. denser than sea water is known as estuarine. Ex:
1) It is formed when river water is denser than sea Narmada River which flows westwards and joins
water. Arabian sea and forms estuary. Bird-Foot delta is
2) It is formed when river water is as dense as sea formed when the river water is less dense than
water. the sea water. Ex: Mississippi delta.
3) It is known as growing delta.
Choose the correct statement/s from the following
codes.
a) 1 only b) 2 only
c) 1, 3 only d) 2, 3 only

183. Troposphere being the weather layer is the most Answer : (a)
important layer. Consider the statements
Because of the strong convection in the Equatorial
regarding this weather layer. region, the tropopause is pushed upward there by
1) The height of the troposphere increases from increasing the height of troposphere in the
Equator to the Poles. Equators while it is not the case in the
2) The height of the troposphere is comparatively
Poles.
more during the summer season than in the winter
season. During summer season, the air gets heated up
and creates low pressure on the ground thus
3) The height of the troposphere doesn't get affected
increasing the pressure gradient which propels
by the change in seasons.
the air movements upwards and in turn pushing
Choose the correct statement/s from the following
the tropopause upwards.
code.
a) 2 only b) 1, 3 only
c) 2, 3 only d) 3 only

184. Generally the incoming Solar radiations are Answer : (a)


absorbed, scattered and reflected in the Atmospheric gases especially green house gases
atmosphere. Based on the above statement
absorb radiations and heat up the atmosphere.
which of the following is/are correctly matched?
Dust particles and haze are largely involved in the
1) Clouds — reflection. scattering of the light.
2) Atmospheric gases --- scattering.
3) Dust particles, haze — absorption.
Choose the correct code from the codes below.
a) 1 only b) 1, 2 only
c) 1, 3 only d) All the above

64
185. The atmospheric temperature significantly Answer : (d)
varies in its distribution pattern. The Apart from the three mentioned, the variability is also
factors that are the reason for this the effect of prevailing winds, cloud cover and altitude.
variability include
1) Sign of latitude.
2) Land and water distribution.
3) Oceanic currents.
Choose the correct reason from the following
codes.
a) 1, 2 only b) 2, 3 only
c) 1, 3 only d) All the above

186. The extreme cold winters in the Central low Answer : (b)
lands of U.S.A is due to a particular polar Loo is a local seasonal wind that blows in summers in
wind. the Northern India.
Which is that wind? Harmattan wind blows in Saharan region. It blows
a) Loo from the Northeast and from East to
West in the Saharan region.
b) Blizzard
Chinook blows down the slope from the Rocky
c) Harmattan
mountain in the U.S.A. The winds descending from the
d) Chinook mountains get heated up adiabatically and reduce the
severity of the winters in the foot hill region.

187. Coral reefs destruction is a major concern Answer : (c)


in the backdrop of Climate Change. They are confined to shallow waters.
Consider the statements regarding the
coral reefs.
1) Coral reefs are example of a symbiotic relation
between coral polyps and photosynthetic
algae.
2) Coral reefs are confined to deep waters.
3) Coral reefs are marine equivalent of tropical
rain forest.
Which of the statement/s is/are true?
a) 1, 2 only b) 2, 3 only
c) 1, 3 only d) All the above

65
188. Consider the following statements. Answer : (c)
1) Pelagic waters are the surface waters between
two shorelines extensive upto the depth of 500
meters.
2) Benthic waters extend from the depth of 500
meters to the base of the ocean.
Which of the following statement/s is/are true?
a) 1 only b) 2 only
c) Both the above d) None of the above

189. In which part of the ocean does the typical Answer : (a)
example of reversal of ocean current can be In the Northern part of Indian Ocean, the ocean
observed which is guided by the planetary currents are clearly guided by the planetary winds.
wind system? There is complete reversal of the ocean currents
a) Northern part of Indian Ocean direction right from the African Eastern coast to the
Indian coast. Ex: Somalian current flows from North to
b) Southern part of Indian Ocean
South in the African East coast during winters when
c) North Atlantic Ocean there is off shore North East Trade wind. The same
d) South Pacific Ocean current reverses its direction when there is South West
Monsoon wind. It flows from South to North.

190. Consider the statements regarding the Answer : (b)


Mediterranean climate. During summers this climatic region comes under the
1) During summers this climatic region comes influence of Off-shore Trade winds.
under the influence of On-shore Westerly During winters this climatic region comes under the
wind. influence of On-shore Westerly wind.
2) During winters this climatic region comes
under the influence of Off-shore Trade wind.
3) Atmospheric stability exists over the region
during summers because of anti-cyclonic
conditions.
Which of the statement/s is/are correct?
a) 1, 2 only b) 3 only
c) All the above d) None of the above

66
191. Atacama desert is one of the driest deserts Answer : (c)
of the world. Apart from other reasons
there is a particular current which has
made it a driest desert. Which is that
current?
a) Benguela current.
b) Okhotsk current.
c) Peruvian current.
d) None of the above.

192. Frank Joseph land of Russia makes the Answer : (b)


northern most limit of summer vegetation No cyclonic depression exists because of very low
in Tundra climate. Which is the type of temperature and also weak pressure gradient.
precipitation that occurs in this region that
Convectional precipitation to occur, the air on the land
supports this vegetation?
surface should be heated to larger extent so that it
a) A very low cyclonic depression that causes light becomes lighter thus rises and condenses to give
rainfall in the form of snow fall. precipitation. But very low temperature in the higher
b) Frontal precipitation that causes precipitation latitudes causes no convectional precipitation.
in the form of snow fall.
c) Convectional type of precipitation.
d) Both 1 and 2

193. Desert is mainly located in the tropical Answer : (c)


West margins of the continents. What is/are Second statement: it is Off-shore winds and not On-
the possible reason/s? shore winds the blows over this region. This winds
1) Existence of the perennial high pressure over carry no moisture thus causes no precipitation.
this region. Ex of cold currents: canaries current in the Atlantic
2) Prevailing On-shore winds throughout the East coast, West Australian current in the West
year. continental margins of Australia etc.
3) Cold currents on the continental west margins
add to the atmospheric stability of the region.
Choose the correct code from the following.
a) 1 only b) 1, 2 only
c) 1, 3 only d) 2, 3 only

67
194. Amongst the following the river that passes Answer : (c)
through most number of countries? The Danube River touches or passes through ten
(a) Zambezi (b) River Severn countries, more than any other river. The Danube
(c) Danube (d) Missisippi begins in Germany and encounters Austria, Slovakia,
Hungary, Croatia, Serbia, Romania, Bulgaria, Moldova,
and Ukraine. It eventually flows into the Black
Sea.

195. Snow line is a line above which no Answer : (a)


vegetation grows on high altitude Snowline is a function of altitude and latitude. Higher
mountains because of permanent snow is the latitude and altitude lower is the snowline.
cover. This snow line is higher for Eastern Eastern Himalaya’s altitude as well as latitudinal
Himalayas compared to that of Western location is lower compared to Western Himalayas. Due
Himalayas. What are the reasons? to higher altitude as well as latitude the temperature is
Consider the following statements much less in the Western Himalaya and as a
1) Eastern Himalayas are near to the Equator. consequence the snowline in the Western Himalaya is
2) Western Himalayas are more continental in at a lower altitude than in the Eastern Himalaya.
their location thus has lesser maritime Southwest monsoon winds bring more rainfall to
influence. Eastern Himalayas than compared to Western
3) Southwest monsoon winds have the Himalayas. Thus monsoon winds can have moderating
moderating effect on the Western Himalayas. effect on Eastern Himalayas and in turn increasing the
Choose the correct code altitude of snowline. Thus 3rd statement is wrong.
(a) 1, 2 only (b) 1, 3 only
(c) 2, 3 only (d) All the above

196. The frequency of the cyclones is lower in Answer : (c)


Arabian sea when compared to Bay of Bay of Bengal being a closed and a smaller water body,
Bengal. This is because it has higher surface temperature compared to
1) Arabian sea surface temperature is higher Arabian sea.
compared to that of Bay of Bengal. North Western region of Pacific ocean has the highest
2) Bay of Bengal receives the remnants of the proportion of global tropical cyclones. These cyclones
typhoons originated in Northwestern Pacific which originate in the Pacific ocean start moving
ocean. towards south western direction and finally reach Bay
3) Arabian sea receives the remnants of cyclones of Bengal. But by the time they reach, they almost lose
from Bay of Bengal. their energy and only remnants of cyclones reach Bay
Choose the correct code from the following of Bengal. Similarly the cyclones originated in Bay of
(a) 1, 2 only (b) 1, 3 only Bengal reach Arabian sea but only remnants of
(c) 2, 3 only (d) All the above cyclones after shedding their energy while traversing
over the peninsular landmass.

68
197. Coal bed methane is released during coal Answer : (b)
mining and is generally recommended to CBM doesn’t cause any as such fire hazards if left un
extract it rather than letting it evaporate
extracted.
into the atmosphere when untapped
CBM once extracted and used as a source of energy it
because
will be less harmful to environment because the
1. CBM if is not extracted can cause fire hazards
burning of CBM releases majorly carbon dioxide. But if
while mining deep areas.
it is not extracted it will escape into atmosphere and
2. CBM if allowed to escape into atmosphere, will
get converted into carbon monoxide which is more
get converted into carbon monoxide which is a
potent green house gas than carbon dioxide.
potent green house gas.
Which of the statements is/are the correct
reason/s?
(a) 1 only (b) 2 only
(c) Neither 1 nor 2 (d) Both 1 and 2

198. Khadar soils are preferred over Bhangar Answer : (a)


soils for crop cultivation because Bhangar soils are rich in kankarie calcareous deposits
1. Khadar soil are composed of fine particles than like calcium carbonate. Second statement is wrong and
that of Bhangar soil. Bangar soils are rich in kankar nodules ie calcareous
2. Bhangar soils lack in kankar nodules. deposits. Infactkankar deposits improves soil fertility.
Choose the correct reason from the codes below. Also that in Khader soils nutrients are renewed every
(a) 1 only year from flood waters and thus more fertile than
(b) 2 only Bangar soils which is rarely replenished with nutrients.
(c) Both
(d) none of the above

199. Consider the statements regarding red and Answer : (c)


yellow soils.
This type of soils get red colour when iron is diffused
1. Soil looks yellowish when iron diffused in the in the crystalline and metamorphic rocks. The same
soil gets hydrated. soil turns yellow when the iron diffused gets turned
2. Red and yellow soils are mainly found in the into hydrated form.
deltaic regions especially in the Eastern Indian
These soils occur generally in the drier regions of
coast.
Eastern and Southern India and not in the deltaic
3. Less fertile coarser soils are found in the
regions.
uplands while fertile fine soils occur in the low
lying regions.
Which are the correct statements?
(a) 1 only (b) 1, 2 only
(c) 1, 3 only (d) 2, 3 only

69
200. Ragi, a millet is currently encouraged to include in Answer : (d)
the diet to supplement nutrients to counter the
Among minor producers even Jharkhand,
malnutrition that is haunting the country.
Jammu Kashmir are also involved apart from
Consider the following statements Ragi.
the states mentioned in the 2nd statement.
1) Ragi is rich in iron, calcium, roughphage and is mainly
grown in dry regions. Ragi production and consumption is highest in
2) Among minor producers Sikkim, Arunachal Pradesh, Bengaluru-Mysuru table region.
Uttarakhand are included.
3) It is grown as a major crop in Bengaluru-Mysuru table
land from where the country's majority of Ragi
production comes from.
Choose the correct statements from the code below.
(a) 1, 2 only (b) 2, 3 only
(c) 1, 3 only (d) All the above

201. Although green revolution brought a great success Answer : (c)


for our country during later 1960's it also brought in
Inter cropping disparity because it was food
few negatives. What are those negatives?
grains which got impetus especially wheat and
1) Inter crop disparity.
many other crops were neglected eg: pulses.
2) Inter personal inequalities.
3) Environmental pollution. Largely green revolution favoured large
4) Regional economic disparity. farmers than small and marginal farmers
5) Unemployment. because green revolution needed some initial
6) Self sufficiency in all types of agricultural crop investments for mechanization, high yielding
production. seeds etc while small and marginal farmers
could hardly afford such expenditures.
Choose the correct code from the following.
(a) 1, 2, 3 only (b) 1 , 3, 4, 5 only With unbalanced fertilizers usage it led to soil
(c) 1, 2, 3, 4, 5 only (d) 1, 2, 3, 4, 6 only fertility decline and also crops were grown in
such areas where those were not supposed to
be grown like rice crop which was started in
Haryana and Punjab because of Green
revolution.
Green revolution mainly benefitted North West
India and coastal areas in Eastern India.
Thus agricultural income increased only in
those regions where Green revolution was
brought and thus increasing regional disparity
in terms of economy.
Mechanization was supported and encouraged
to get the better benefits of Green revolution
thus resulting into unemployment. Self
sufficiency was brought only in food grains like
rice wheat etc, and not in every crop.

70
202. Consider the statements regarding Earth's interior Answer : (b)
and seismic waves.
Primary waves (P-waves) are compressional
1) Primary waves totally disappear in the core region
waves that are longitudinal in nature. These
revealing that outer core is made up of liquid.
waves can travel through any type of material,
2) Secondary waves show changes in its speed as it travels
including fluids, and can travel at nearly twice
from magma chamber to lithosphere region indicating
the speed of S waves. Thus will not disappear
difference in density.
in the core. But S waves do disappear in the
3) The speed of secondary waves is highest inside the
core region.
Earth's Core region indicating highest density in the Core
region. Secondary waves (S-waves) are shear waves
Choose the correct statement/s from the code below. that are transverse in nature. S-waves can
a) 1, 2 only b) 2 only travel only through solids, as fluids (liquids and
c) 1, 3only d) 2, 3 only gases) do not support shear stresses. S-waves
are slower than P-waves, and speeds are
typically around 60% of that of P-waves in any
given material. S waves speed varies with
density just like P waves and this speed
increases with increasing density. Since there is
density difference between lithosphere and
magma chamber the speed of the S waves do
change.
S waves do not enter the Core region as the
outer core is made up of liquid.

71
203. Consider the statements regarding Answer : (c)
different types of plates in 'Plate Tectonic
Divergent plate boundary or constructive plate
Theory'. boundary:
1) Destructive plate boundary is one in which A tectonic boundary where two plates are moving
continuous upwelling of molten material i.e. away from each other and new crust is forming from
lava takes place. magma that rises to the Earth's surface between the
2) Divergent plate boundary is one which two two plates. The middle of the Red Sea and the mid-
plates slide past one another thus neither ocean ridge (running the length of the Atlantic Ocean)
creating nor destroying continental landmass. are divergent plate boundaries.
3) Convergent plate boundary is one in which one Convergent plate boundary or destructive plate
plate overrides the other plate. boundary:

Choose the correct statement/s from the following It is a tectonic boundary where two plates move
towards each other. If thetwo plates are of equal
code.
density, they usually push up against each other,
a) 1, 2 only b) 2 only
forming a mountain chain. If they are of unequal
c) 3 only d) All the density, one plate usually sinks beneath the other in a
above subduction zone.
The western coast of SouthAmerica and the Himalayan
Mountains are convergent plate boundaries. Also
called active margin , collision zone.
Transform plate boundary:
Transform plate boundary is one in which two plates
slide past one another thus neither creating nor
destroying continental landmass.

204. Every relief can be classified into different Answer : (a)


types of reliefs. nd
2 Order reliefs are the one formed because of
Match the following. endogenic events like volcano, earthquake or plate
st
1) 1 Order relief --- continental plates. movents either constructive or destructive plate
2) 2
nd
Order relief --- plains and deltas. boundaries. Ex: fold mountains, block mountains etc.
rd
3)
rd
3 Order relief --- fold mountains. 3 Order reliefs are the formed because of exogenic
Choose the correctly matched pair from the events. Ex: flood plains, deltas etc.
following codes.
a) 1 only b) 1, 2 only
c) 1, 3 only d) None of the above

72
205. Consider the following Answer : (b)
1. These clouds are one of the highest clouds in
the atmosphere
2. They are also called thunderheads and
produce rains, thunder and lightning
Select the correct answer based on above
statements
(a) Stratocumulus (b) Cumulonimbus
(c) Nimbostratus (d) Cirrocumulus

206. Which of the statements are true regarding Answer : (d)


Equatorial climatic conditions? The climatic region is extensive from 10degree South
1) It is known as latitudinal climate. to 10degree North. And this type of climatic condition
2) Low range of temperature is mainly due to the is confined to only this latitudinal expanse. So it is also
equatorial air mass. termed as latitudinal climate. Equatorial air mass
3) There is no seasonality in this region or there keeps the temperature moderate in the Equatorial
are no distinguished seasons unlike other region.
climatic regions.
Which among the following statement/s is/are
true?
a) 3 only b) 2, 3 only
c) 1, 2 only d) All the above

207. Natural levees are depositional landforms Answer : (c)


on either side of the banks of a river. What Levee forming regions are good for agricultural
are the disadvantages of natural levees. purposes as its soil is renewed every year thus
1) Levees if breached can cause catastrophic imparting fertility.
floods. As the levee limits the flow of river water within its
2) Levee forming regions cannot be used for boundaries the river sheds its sediments only in the
agricultural purposes. river basin thus making river basins narrower and
3) Levee formation leads to sedimentation of shallow thus increasing the risks of floods.
river basins.
Choose the correct disadvantage/s from the
following codes.
a) 1, 2 only b) 2, 3 only
c) 1, 3 only d) All the above

73
208. Temperature anomaly is the deviation of Answer : (a)
temperature either more or less than the Negative anomaly cannot be seen in higher
average of the surrounding region. latitudes over the waters during day time rather
Consider the statements regarding the same. than night time.
1) Positive anomaly is the record of higher
temperature than the normal which is identified
over the land in lower latitudes during day time.
2) Negative normally is the record of lower
temperature than the normal which is identified
over the sea in higher latitudes during night time.
Choose the correct statement/s from the following.
a) 1 only b) 2 only
c) Both the above d) None of the above
209. Consider the statements regarding earthquake Answer : (d)
(seismology).
Primary waves (P-waves) are compressional
1) Primary waves are analogous to sound waves and it waves that are longitudinal in nature. These
can travel through liquid medium. waves can travel through any type of material,
2) Secondary waves are analogous to water ripples and including fluids, and can travel at nearly twice the
it cannot pass through liquid medium. speed of Secondary waves.
3) Surface waves are most destructive and slowest Secondary waves (S-waves) are shear waves that
among the three waves (primary, secondary, surface are transverse in nature. S-waves can travel only
waves). through solids, as fluids (liquids and gases) do not
Choose the incorrect statement/s from the following support shear stresses.
codes.
a) 1, 2 only b) 3 only
c) 2, 3 only d) None of the above

210. Enrich plantation is something in which a single Answer : (b)


commercially valuable species is extensively
Teak monoculture majorly impacted South India.
planted in a region and other local species are
eliminated.
Consider the statements regarding Enrich
plantation.
1) Teak monoculture has damaged the natural forest
majorly in Northeast India which was earlier
introduced by British.
2) Chir, Pine plantations in the Himalayas have replaced
the Himalayan Oak and Rhododendron forests.
Choose the correct statements from the code below.
a) 1 only b) 2 only
c) Both d) None of the above

74
211. Consider the following statements regarding a Answer : (c)
particular soil Although first two statements i.e. 1 and 2 do
1. Found in the summits of Western Ghats, Eastern comply with mountain soil and forest soil, the
Ghats, Vindhyas, Sathpuras and Malwa plateau. third statement is unique for laterite soil.
2. Subjected to intense leaching and thus not fit for
agriculture unless manures and fertilizers are used.
3. It is the end product of weathering process and thus
it is indefinitely durable.
Which is the soil that is spoken about in the above
statements?
(a) Forest soil (b) Mountain Soil
(c) Laterite soil (d) Red soil
212. Imagine you are travelling from New Delhi Answer : (b)
towards North. If you go vertically upwards If one travels vertically upward, he/she will not
towards Jammu and Kashmir, select the correct cross Jhelum as it flows North-
sequence in which you will cross these rivers West of J&K. Other rivers will come in the
(a) Ravi-Beas-Ravi-Satluj-Jhelum sequence as mentioned in option (b).
(b) Satluj-Beas-Ravi-Chenab-Indus
(c) Satluj-Beas-Ravi-Chenab-Jhelum
(d) Ravi-Beas-Ravi-Satluj-Indus

213. The soil salinization occurred due to irrigation is Answer : (a)


a concern in drier regions and not in humid
regions. Consider the following statements
1. In humid regions, there is enough precipitation to
leach (wash) the salts below the plant root zone.
2. In drier regions, higher temperature leads to higher
evaporation of water resulting into leaving behind
of salt on the surface of the soil.
3. Soil salinization occurring due to irrigation can
equally impact both humid areas as well as drier
areas.
Choose the correct statement from the code given
below
(a) 1, 2 only (b) 1 only
(c) 2 only (d) 3 only

75
214. Sugarcane grown in subtropical region has lower Answer : (a)
productivity than that is grown in tropical First two statements are self explanatory.
regions. A higher diurnal temperature and low humidity
Consider the following statements about are required for a better productivity. Thus these
subtropical region i.e. Northern India which has two factors are not reducing the productivity.
low productivity in sugarcane production.
1) Frost formations.
2) Floods and water logging.
3) High diurnal temperature.
4) Low humidity.
Choose the correct reason/s from the codes below.
(a) 1, 2 only (b) 2, 3 only
(c) 3, 4 only (d) 1, 2, 3 only
215. Winter solstice for Northern Hemisphere Answer : (b)
nd st
generally is on 22 or 21 December every year. Winter solstice occurs for the Northern
Consider the statements regarding winter Hemisphere in December and for the Southern
solstice in Northern Hemisphere. Hemisphere in June.
1) The circle of illumination excludes the North Pole In the second statement it should have been 66.5
region from Sunlight. degree North to 90 degree North latitude and not
2) From 66.5 degree South to 90 degree South, the South latitudes.
Sun remains below the horizon the entire day.
3) Arctic Circle is the Southernmost parallel in the
Northern Hemisphere that experiences a 24 hour
period of darkness on this day.
Choose the correct statements from the codes below.
a) 1 only b) 1, 3 only
c) 2 only d) None of the above

216. The atmosphere is a gaseous envelope which Answer : (b)


surrounds Earth's surface. Consider the Rotation of earth induces centrifugal force which
following statements regarding the Earth's pushes any object away from the Earth’s surface.
atmosphere. Thus rotation doesn’t help atmosphere to bind to
1) Rotation of Earth helps in binding of atmosphere to Earth’s surface.
Earth's surface. Atmosphere filters the harmful rays from the Sun
2) It sustains life on the Earth by not allowing Earth to and also helps in controlling the temperature so
become too hot or too cold. that life can be sustained.
3) Content of water vapour increases from the Content of water vapour is highest near to surface
equator towards the poles. of the Earth ie upto some 5 kms from the surface,
Choose the correct statement/s from the following beyond that it reduces drastically.
code.
a) 1, 2 only b) 2 only
c) 2, 3 only d) All the above

76
217. Temperature in the Stratosphere increases Answer : (a)
because of the presence of Ozone. This Ozone
_______ the sunlight
a) Absorbs
b) Scatters
c) Reflects
d) None of the above

218. Although laterite soils are formed in regions Answer : (c)


with high rainfall and high temperature, it Intense leaching is caused by heavy rainfall which
requires good dosage of manures. What are the leaches silica and makes soil a bit acidic.
reasons? High rainfall also inhibits humus formation with
1. Intense leaching because of high rainfall. decreased bacterial activity.

2. lack or less availability of humus.


Choose the correct code from the following
(a) 1 only
(b) 2 only
(c) Both
(d) None of the above

219. Laterite soils and Forest soils are acidic in Answer : (a)
nature. What is the reason for this? Water from rain, flooding, or other sources seeps
(a) High rainfalls leach away the cations into sub soil into the ground, during which it can dissolve
region. chemicals and carry them into the underground
water supply. Thus also causing ground water
(b) Trees and plants absorb the cations necessary for
contamination.
their development.
(c) Cations are washed away along with running water
and drained into sea/ocean.
(d) None of the above reasons are correct.

77
220. Tea best grows in Assam, Darjeeling and Jalpaiguri Answer : (b)
districts of West Bengal. What is/are the Showers are almost evenly distributed
uniqueness of this region which suits this crop? throughout the year with very low dry spell.
1) It has warm and humid climate. Well drained gentle sloped soils are required
and rugged topography is not suitable for tea
2) Showers are mainly concentrated in 4-5 months i.e.
plantations.
June to October in any year.
3) Deep, fertile and well drained soil.
4) Rugged topography of the relief near the Himalayan
foot hills.
Choose the correct code.
(a) 1, 2, 3 only (b) 1, 3 only
(c) 1, 3, 4 only (d) All the above

221. Himalayan yew is a plant found in Himachal Answer : (b)


Pradesh and Arunachal Pradesh was in news
A chemical compound called ‘ taxol ‘ obtained
recently. from the plant is used as an effective anticancer
Consider the following statements regarding the drug.
plant.
1) It is a medicinal plant.
2) A chemical compound called 'Taxol 'obtained from
the plant is used as an effective anti-TB drug.
Choose the incorrect statements from code below.
(a) 1 only (b) 2 only
(c) Both are true (d) None of the above

222. There are different discontinuities within the Answer : (a)


Earth's internal structure. Which are the
Mohorvic discontinuity is the boundary between
discontinuities that have been incorrectly
Crust and Mantle.
matched with its/their respective locations?
Wiechert-Gutenberg discontinuity is the
1) Mohorvic discontinuity boundary between Core and boundary between Core and Mantle.
Mantle.
2) Wiechert-Gutenberg discontinuity boundary
between Crust and Mantle.
3) Conrad discontinuity - located at a depth of 15-20kms
from the Earth's surface.
Choose the correct code from the following.
a) 1, 2 only b) 2, 3 only
c) 3 only d) 2 only

78
223. Weathering is the basic and first step in soil Answer : (a)
formation. Consider the statements regarding Steepness increases the weathering rate. The
weathering. weathered rocks are easily removed by
1) The rate of weathering increases with increasing gravity, flowing water, moving wind etc if the
steepness of the slope. steepness is higher thus exposing new rocks
2) Mechanical weathering is higher in humid regions than for weathering thereby increasing its rate.
in drier regions. Mechanical weathering is higher in drier
3) Chemical weathering is higher in drier regions than in regions
humid regions. as there exists higher diurnal temperature.
Few minerals are water soluble like calcium
Choose the correct statement/s from the codes below.
carbonate etc thus humid region favours the
a) 1 only
chemical weathering. Moreover water is the
b) 2, 3 only
main agent of chemical weathering in the
c) 1, 3 only
rocks.
d) All the above.

224. During monsoon season the ITCZ (inter tropical Answer : (d)
convergence zone) shifts almost upto 25 degree
Inter tropical convergence zone is termed as a
North latitude over the Indian subcontinent while it thermal equator and is the zone where the
rarely crosses 15 degree North latitude in other
Northeast and Southeast trade winds meet.
regions. This is because This ITCZ shifts Northwards upto 25 degree
1) Vast landmass of Eurasia located to the North of the North latitude during the months of June-July.
Equator. This is facilitated by all the above points
2) Intense low pressure over the Northwestern region of mentioned.
Indian subcontinent and over the vast, high altitude Westerly jet streams: These shifts southward
Tibetan plateau pulls ITCZ Northwards. during winters because of the southern shift
3) Shifting of Westerly jet streams to the North of Tibetan of entire pressure belts by some 10 degree
plateauat the end of the month of May. latitudes which is in turn because of Southern
Choose the correct code from the following shift of Sun towards Tropic of Capricorn
(a) 1, 2 only (b) 2, 3 only during winters. These jet streams because of
(c) 1, 3 only (d) All of the above vast Tibetan plateau,get bifurcated into 2
parts, with one part traversing south of
Himalyan mountains and the other to the
north of Tibetan plateau. With the advent of
summers and gradual shift of Sun towards
Tropic of Cancer, the part of the jet streams to
the South of Himalayas gets weakened. At the
end of the month of May the Westerly jet
streams completely shifts to the North of
Himalayas and thus becomes stronger over
the Central China and also Japan. This
Northern shift facilitates the Northward shift
of ITCZ.

79
225. Consider the following statements which describe the Answer : (a)
properties of a particular metal
1. It as a valuable metal of which the major ore is Wolfram.
2. It is self hardening which it imparts to the steel when
alloyed with the steel.
3. Steel when alloyed with this metal, is used in
manufacturing of ammunitions, armour plates, heavy
guns etc.
4. Its alloys are heat resistant, corrosion resistant and
hardfacing.
Choose the correct metal from the options
(a) Tungsten (b) Chromium
(c) Copper (d) Titanium

226. The productivity in the Indian agriculture has Answer : (d)


stagnated since 1990's although the usage of Unscientific usage is the main reason ie
fertilizers has considerably increased since then.
unbalanced dosage of nitrogen, phosphorous,
What is/are the possible reason/s? potassium. In the ideal condition it should be
(a) Unscientific usage of fertilizers. 4:2:1. But in India it is completely unbalanced.
(b) Agriculture productivity in India has reached a zenith Based on region the ratio of N:P:K varies in a
position and any further increase in the productivity big way eg: all India it is 8.2:3.2:1, for Punjab it
cannot come through increase in the usage of fertilizers. is 61.9:19.3:1, for Haryana it is 61.4:18.7:1.
(c) Increasing monsoon vagaries because of Climate Change. This can have negative impact like decreasing
(d) Both 1 and 3 the soil fertility and thus reducing the yield.
Monsoon vagaries definitely has increased
with time.

227. The trees found in Western side of Western Ghats Answer : (d)
and in the Northeast region generally don't shed
Prolonged dry season and severe winters do
their leaves i.e they are evergreen.
result in shedding of leaves by trees.
What are the possible reasons? Tropical deciduous trees shed their leaves
1) These regions do not have prolonged dry season. during dry season because of excess
2) These regions don't have severe winters. evaporation and temperate deciduous trees
3) These regions have the humidity level beyond 75% shed their leaves during severe winters.
almost throughout the year.
Choose the correct reason/s from the codes below.
a) 1 only b) 2 only
c) 1, 2 only d) All the above

80
228. Consider the following statements Answer : (c)
(1) The West flowing rivers contains very less amounts of
Silt and due to its fast speed it cannot make delta.
(2) Rivers arriving into a sea with high tidal range will not
form delta because the changes in the tidal area will
wash away the sediments brought by the river.
(3) Western rivers flows in the fault region created by the
mountains Vindhya and Satpura which are rocky and
devoid of any alluvial material. Hence no Delta
formation
Select the correct statement/s
(a) Only 1 (b) 1 and 2
(c) 1, 2 and 3 (d) Only 2

229. The best way to improve crop production in India is to Answer : (d)
increase the crop intensity i.e. by increasing gross
In fact black soil is an added advantage in
sown area.
improving the crop intensity as it requires
Which is not the hindering factor involved in very less irrigated or rainfall water as it has a
improving the crop intensity? very good capability to retain moisture.
a) Infertile soil. The usage of manures and fertilizers per
b) Deficiency in moisture, especially in rain fed region. hectare is quite low in India compared to
c) Insufficient usage of manures and fertilizers. other countries like China, Japan etc.
d) Large tracts of black soil.

230. Central Zoo Authority, a body set up in 1992 for the Answer : (a)
oversight of zoos. Consider the following It does have the power to regulate the trade
statements regarding CZA. of endangered species. Apart from that it also
1) It is set up under the Wild Life Protection Act, 1972. has the power to regulate and approve the
2) It is an affiliated member of World Association of Zoos exchange of animals between Indian and
and Aquarium. foreign zoos.
3) It doesn't have the power to regulate the trade of
endangered species.
Choose the correct statements from the following code.
(a) 1, 2 only (b) 2, 3 only
(c) 1, 3 only (d) All the above

81
231. Few forests in India are protected under the name Answer : (c)
of Sacred Grooves. Consider the following Prior to 2002 these forest regions were not
statements regarding Sacred Grooves. recognized under any of the existing laws. But
1) It is a nature worship which is an age old belief based in 2002 an amendment was brought in
on the premise that all the creations of the mother Wildlife Protection Act, 1972 to include Sacred
Nature have to be protected. Grooves under the act.
2) It is recognized under Wildlife Protection Act, 1972.
Choose the correct statement/s from the following code.
a) 1 only b) 2 only
c) Both d) None of the above

232. A particular ocean has a broad shape of 'S' in the Answer : (a)
North South orientation. Which is that ocean?
a) Atlantic Ocean.
b) Pacific Ocean.
c) Indian Ocean.
d) Southern Ocean.
233. Continental Drift theory by Wegener changed the Answer : (b)
perceptions about the Earth's crust. Continental Drift theory by Wegener tried to
Consider the statements regarding this theory. solve the problem of location of young fold
1) Sialic masses used to float over the Sima without any mountains.
resistance being offered by Sima. nd
2 statement: panthalasa was the name given
2) Earlier all land masses were united to form a massive to single vast ocean and pangaea was the
landmass known as 'Panthalasa'. name given to united massive landmass.
3) Ocean floors were considered to be stable and never
moved.
Choose the correct statement/s from the following.
a) 1 only b) 1, 3 only
c) 2, 3 only d) None of the above

234. Consider the statements regarding the cyclones. Answer : (d)


1) In a cyclone, the air circulation is anti-clockwise in the
Northern Hemisphere and clockwise in the Southern
Hemisphere.
2) Tropical cyclones being embedded in the Trade winds,
in general move from East to West.
3) Temperate cyclones being embedded in the Westerlies,
in general move from West to East.
Choose the correct statement/s from the following codes.
a) 1, 2 only b) 2, 3 only
c) 1, 3 only d) All the above

82
235. Consider the following Dams Answer : (b)
1. Barna Dam : Chambal River Barna Dam: Barna River in MP
2. Pawna Dam : Maval River Pawna Dam: Maval River in Maharashtra
3. Maithoon Dam : Barakar River Maithoon Dam: Barakar River in Jharkhand
4. Gandhi Sagar Dam : Chambal River Gandhi Sagar Dam: Chambal River in MP
Select the correct options
(a) 1, 2 and 3
(b) 3 and 4
(c) 1, 2, 3 and 4
(d) 1, 3 and 4

236. Plasma is one of the four fundamental Answer : (a)


states of matter. Consider the following It neither has volume nor has shape.
statements. The matter in Sun and other stars in the universe is
1) It doesn't have volume but has shape. made up of plasma matter, hence most abundant.
2) It is the most abundant form of matter. Electromagnetic phenomenon can be used to ionize gas
3) Gas or air can be turned into plasma when particles.
artificially ionized.
Choose the correct statements from the codes
below.
a) 3 only
b) 1, 3 only
c) 2, 3 only
d) 1 only

237. Paleomagnetism is the past magnetism of Answer : (a)


Earth stored in a rock.
Increasing reliability of the rocks: igneous >
Which among the following type of rocks sedimentary > metamorphic.
provides most reliable readings of
magnetism at a particular time?
a) Igneous rocks.
b) Sedimentary rocks.
c) Metamorphic rocks.
d) All the above rock types are equally reliable

83
238. Continental rise is a feature found on the Answer : (a)
boundary of the continental slope. Consider the Second statement: It is not found in the active
statements regarding the continental rise.
boundary because these structures will be
1) Continental rise are comprised of the finer textured destructed during the formation of trenches in
smaller materials. the active plate boundary. Thus found only in
2) Continental rise are found only in the active boundary the passive boundaries.
continental slope.
Which of the following statement/s is/are correct?
a) 1 only
b) 2 only
c) Both the above
d) None of the above

239. Consider the statements regarding insolation Answer : (c)


which is nothing but the radiant energy received
Solar rays will have to travel longer distance to
from the Sun in the form of heat and light. reach the polar region than the Equatorial
1) Insolation received on the surface of the Earth regions thus large amount of energy is lost due
decreases from Equator to the Poles. to scattering, diffraction, reflection etc. thus
2) During Summer Solstice, maximum insolation higher insolation is experienced in the
received at the ground surface is between the 30 to equatorial region.
40 degree South latitudes. Similarly the insolation is higher in the outer
3) Insolation received is comparatively higher in the limits of the atmosphere.
outer limit of the atmosphere than that is on the In the second statement, the latitudes should
surface of the Earth. have been North rather than South.
Choose the correct statement/s from the following
codes.
a) 1, 2 only b) 2, 3 only
c) 1, 3 only d) All the above

240. Consider the following statements regarding the Answer : (c)


production and reserves of copper in India.
Rajasthan although has the highest reserves, it
1. Rajasthan has the largest reserves of copper and is is not the highest producer. Madhya Pradesh is
also the highest producer of copper in India. the highest producer currently.
2. India is self sufficient in copper production and also The production of copper in India always falls
at times it has also exported copper to countries like short of demand and thus we import copper
Japan, South Korea etc. from countries like US, Japan, Canada etc.
3. Production of copper is a tedious job especially in Third statement is true because in India the
India as the copper ore found in India as it is of the metal content in the ore is quite low compared
lower grade compared to International grade of the to international average. The metal content is
ore. less than 1% in India while the international
Which are the correct statements? average stands at 2.5%.
(a) 1 only (b) 1, 3 only
(c) 2, 3 only (d) 3 only

84
241. Consider the following statements regarding the Answer : (b)
atomic minerals. Australia has the highest reserves of uranium
1. Uranium occurs both in the earth's crust as well as and not the US.
sea water.
2. India has one of the richest reserves of thorium in the
world.
3. US have the highest reserves of uranium in the world.
Which of the above statements are correct?
(a) 1 only (b) 1, 2 only
(c) 2, 3 only (d) All the above

242. Kerala has outpaced every other state in the Answer : (b)
country in terms of Rubber production. The
Kerala largely has laterite soil and not red soil.
reasons being
1) It has hot and humid climate with precipitation more
than 200cms annually.
2) Very small dry spell during summers for 2-3 months.
3) Well drained red soil with gentle slope and no
stagnation of water.
Choose the correct reason/s from the code below.
(a) 1 only
(b) 1, 2 only
(c) 1, 3 only
(d) All the above

243. Which of the following area comes under Answer : (c)


earthquake zone V?
Lakshadweep islands - Zone III
1. Northeastern India West coast of Maharashtra - Zone IV
2. Lakshadweep islands Union Territory of Delhi - Zone IV
3. Andaman & Nicobar Islands Sikkim - Zone IV
4. West coast of Maharashtra
5. Union Territory of Delhi
6. Sikkim
a) 1, 2, 3, 4
b) 1, 3, 5
c) 1, 3
d) All the above

85
244. Consider the following statements Answer : (d)
1. Maritime transport is to be administered Maritime transport is to be administered by both the
by Central government. Central and the State governments. While the central
2. All major ports are government government's shipping ministry administers the major
administered ports, the minor and intermediate ports are administered
Which of the above statements are correct? by the relevant departments or ministries in the nine
a) Only 1 coastal states Andhra Pradesh, Odisha, West Bengal, Tamil
b) Only 2 Nadu, Kerala, Karnataka, Goa, Maharashtra and Gujarat.
c) Both All major ports, except one Ennore Port are government
d) None administered. It is the first port in India which is a public
company.

245. Consider the following statements Answer : (c)

1. The subduction of the Nazca tectonic plate


to the east is responsible for the creation
of the volcanic Andes mountain range

2. Volcano Calbuco in southern Chile erupted


for the first time in more than five decades
Which of the above statements are correct?
a) Only 1
b) Only 2
c) Both
d) None

246. Consider the following statements Answer : (c)


1. Tiwa tribe celebrate Yangli festival once in Tiwa is an ethnic group inhabiting the states of Assam and
three years. Meghalaya in Northeast India.
2. Sowing of paddy starts immediately after
the festivals.
Which of the above statement is correct?
a) Only 1
b) Only 2
c) Both
d) None

86
247. Which of the following has highest albedo? Answer : (c)
a) Deciduous forest Fresh snow has the highest albedo. Immediately after
b) Desert a fresh snowfall, up the 90% of the solar radiation can
c) Fresh snow be reflected off the snow.
d) Glacier ice

248. Which of the following is/are not Answer : (d)


characteristics of El nino? Below-average sea surface temperatures across the
a) Increased surface pressure over the Indian east-central Equatorial Pacific.
Ocean, Australia and Indonesia Below-average sea surface temperatures across the
b) Rise in warm air near Peru east-central Equatorial Pacific is a characteristic of La
c) Fall in air pressure over eastern and central Nina which is anti El-Nino.
Pacific ocean and Tahiti.
d) Below-average sea surface temperatures across
the east-central Equatorial Pacific.

249. Which of the following are causes of eruption of Answer : (d)


volcano?
1. Due to diverging or converging tectonic plates
2. High volume magma's upper movement
3. Decrease in pressure of Magma as it rises up
towards the crust
4. Injection of a new batch of magma into an already
filled magma chamber
a) 1, 2
b) 1, 3
c) 1, 2, 3
d) All

250. Consider the following statements Answer : (c)


1. The subduction of the Nazca tectonic plate to the
east is responsible for the creation of the volcanic
Andes mountain range
2. Volcano Calbuco in southern Chile erupted for the
first time in more than five decades
Which of the above statements are correct?
a) Only 1
b) Only 2
c) Both
d) None

87
251. Consider the following statements Answer : (c)
1. Barren Island Volcano is the only confirmed active Dormant volcanoes became quite after eruptions for
volcano in India. some time and there are no indications for further
2. Narcondam Island of the Andaman & Nicobar is eruptions but suddenly they erupt very violently.
example of dormant volcano
Which of the above statements are correct?
a) Only 1
b) Only 2
c) Both
d) None

252. Consider the following statements Answer : (c)


1. Kedarnath is near Chorabari Glacier, the head of
river Mandakin
2. River Mandakina is tributary of the Alaknanda
river.
Which of the above statements is/are correct?
a) Only 1
b) Only 2
c) Both
d) None

253. Consider the following statements Answer : (d)


1. El Nino and La Nina are opposite phases of what is
known as the El Nino - Southern Oscillation (ENSO)
cycle.
2. ENSO can be neutral, warm or cold
Which of the above statements are true?
a) 1, 2
b) 2, 3
c) 1, 3
d) All the above

88
254. Consider the following statements Answer : (b)
1. Strong El nino : Drought in south America Strong El-nino :- Major flooding in South America.
2. La nina : heavy rains over Malaysia, the Philippines, La-Nina:- Heavy rains over Malaysia, the Philippines and
and Indonesia. Indonesia.
Which of the above statements is/are incorrect?
a) Only 1
b) Only 2
c) Both
d) None

255. Consider the following statements Answer : (c)


1. North American monsoon gives rain to west coast
in US
2. East Asian Monsoon gives rain to southern China,
Korea and parts of Japan.
Which of the above statements is/are correct?
a) Only 1
b) Only 2
c) Both
d) None

256. Which of the following is/are triggered by Answer : (d)


earthquake?
1. Tsunami
2. Landslide
3. Volcanic activity
4. Avalanche
a) 1, 2
b) 1, 2, 4
c) 1, 3, 4
d) All

89
257. Which of the following waves travels slowly? Answer : (c)
P-waves travel fastest, at speeds between 4 - 8 km/sec
a) P waves
(14,000 - 28,000 km/h) in the Earth's crust. S-waves
b) S waves travel more slowly, usually at 2.5 - 4 km/sec (9000 -
14,000 km/h) while tsunami travels at 600 miles per
c) Tsunami waves hour (970 km/hr).

d) seismic body wave

258. Consider the following statements Answer : (c)

1. An earthquake's point of initial rupture is called


its focus or hypocenter.

2. The epicenter is the point at ground level directly


above the hypocenter.

Which of the above statements is/are correct?

a) Only 1

b) Only 2

c) Both

d) None

259. What is the correct sequence of arrival of Answer : (a)


seismic waves

a) P waves- S waves .. Love waves - Rayleigh waves

b) Surface waves ... P waves .... S waves

c) P waves ... Surface waves ... S waves

d) S waves ... P waves .... Surface waves

90
260. Which of the following statements are correct? Answer : (d)
1. S- waves travel more quickly through earth's crust P-waves travel more quickly through earth’s crust. They
2. P- waves are more destructive. can travel at nearly twice the speed of S-waves.
a) Only 1 S-waves are more destructive than P-waves because of
their larger amptitude.
b) Only 2
c) Both
d) None

261. Consider the following statements Answer : (c)


1. P - waves are longitudinal waves
2. S - waves are transverse waves.
Which of the above statements is/are correct?
a) Only 1
b) Only 2
c) Both
d) None

262. Which of the following are the causes of eruption Answer : (d)
of volcano?
1. Due to diverging or converging tectonic plates
2. High volume magma's upper movement
3. Decrease in pressure of Magma as it rises up
towards the crust
4. Injection of a new batch of magma into an already
filled magma chamber
a) 1,2
b) 1,3
c) 1, 2,3
d) All

91
263. International date line passes through Answer : (b)
a) 0 degree longitude
b) 180 degree longitude
c) equator
d) Prime meridian

264. Which of the following are conditions for Answer : (d)


occurrence of Loo?
a) Sun moves towards tropic of cancer
b) Heating of subcontinent
c) Continuous belt of low pressure lying between the
subtropical high pressure belts
d) All the above

265. Match the following dust storms with respective Answer : (a)
regions
1. Andhi--------------------- i.West Bengal
2. Norwester----------------- ii. Uttar Pradesh
3. Kalbaisakhi--------------- iii. East India
a) 1- ii, 2- iii, 3-i
b) 1- iii, 2- ii, 3-i
c) 1- ii, 2- i, 3-iii
d) 1- i, 2- ii, 3-iii

92
266. Consider the following statements Answer : (d)
1. Sea breeze refers to the wind system which blows Sea breeze refers to the wind system which blows from
from sea towards the land during night. sea towards the land during the day time. Land breeze is
the wind system which blows from land towards the sea
2. Land breeze is the wind system which blows from
during the night. The primary cause for the sea and land
land towards the sea during day time. breeze is the fact that with similar exposure to the solar
Which of the above statements is/are not correct? energy the land and water body gets heated by
a) Only 1 differently.

b) Only 2
c) Both
d) None

267. Match the following Answer : (d)


1. Chukchi Sea ------------ i.Pacific ocean
2. Timor sea -------------- ii. Artic ocean
3. Marmara sea ------------ iii. Indian Ocean
4. Kora Sea --------------- iv. Atlantic Ocean
a) 1- ii, 2-iv , 3-i, 4- iii
b) 1- iv, 2-iii, 3-i, 4- ii
c) 1- iv, 2-i , 3-iii, 4- ii
d) 1- ii, 2- iii, 3-iv, 4- I

268. Consider the following statements Answer : (a)


1. Gomati is only tributary of Ganga that rises in The Gomti originates from Gomat Taal which formally
plains not in hills known as Fulhaar jheel, near Madho Tanda, Pilibhit,
2. Gomati is right bank tributary of Ganga India. Gomati is left bank tributary of ganga.

Which of the above statements is/are correct?


a) Only 1
b) Only 2
c) Both
d) None

93
269. Consider the following statements Answer : (b)
1. Shipki La pass is in Uttaranchal. Shipki la is a mountain pass and border post on the India-
2. The river Sutlej enters India through this pass. tibet border. The river Sutlej enter India through this
pass. It is located in Kinnaur dist. In the state of H.P.,
Which of the above statements is/are correct?
India and Tibet. The pass in India’s third border post for
a) Only 1 trade with Tibet, after Nathula in Sikkim and Lipulekh in
b) Only 2 Uttarakhand.
c) Both
d) None

270. Consider the following statements Answer : (c)


1. River deltas in eastern ghats causes discontinuous Coastal plains between Western Ghats and the Arabian
hills sea are very narrow. So coastal rivers are short hence do
2. Nilgiri hills connect Western and Eastern Ghats not form delta on western coast.

Which of the above statements is/are correct?


a) Only 1
b) Only 2
c) Both
d) None

271. Consider the following statements Answer : (a)


1. Anamudi highest peak of western ghat
2. It is in Tamilnadu
Which of the above statements is/are correct?
a) Only 1
b) Only 2
c) Both
d) None

94
272. Which of the following pair of rivers is wrongly Answer : (d)
matched with type of delta? Arcuate delta :- Triangle shaped delta.
1. Godavari: Lobate Lobate Detta :- Large amount of sediment with weak or
2. Krishna: Arcuate disorganised ocean currents.
3. Kaveri: Quadrilateral
a) 1,2
b) 2,3
c) 1,3
d) None

273. Consider the following statements Answer : (c)


1. It rains throughout the year brought because of Equitorial Guinea lies between latitude 4°N & 2°S and
low pressure belt created by Doldrums. longitudes 5° & 12° East. Despite its name, no part of
2. Equator passes through Equatorial Guinea country’s territory lies on equator.

3. Volcano Cayambe in Ecuador is the only point on


the Equator with snow cover.
Which of the above statements are true?
a) 1,2
b) 2,3
c) 1,3
d) All

274. Match the following tribes with region Answer : (d)


1. Hausa ---------------- i.SriLanka
2. Tartar --------------- ii.Nigeria
3. Vedda ---------------- iii. Malysia
4. Ainu ----------------- iv. Siberia
5. Semang --------------- v. Japan
a) 1- ii, 2- v, 3-i, 4- iii ,5-iv
b) 1- iv, 2- v, 3-i, 4- ii ,5-iii
c) 1- iv, 2- v, 3-i, 4- ii , 5-iii
d) 1- ii, 2- iv, 3-i, 4- v ,5-iii

95
245. Match the following deserts with region Answer : (b)
1. Chihuahuan Desert ---------- i. Israel
2. Karakum Desert ------------- ii. USA
3. Taklamakan Desert ---------- iii. North America
4. Mojave Desert ------------- iv. Turkmenistan
5. Negev Desert --------------- v. China
a) 1- ii, 2- v, 3-i, 4- iii, 5-iv
b) 1- iii, 2- iv, 3-v, 4- ii, 5-i
c) 1- iv, 2- v, 3-i, 4- ii, 5-iii
d) 1- ii, 2- iv, 3-i, 4- v, 5-iii

276. Which of the following are reasons for formation Answer : (c)
of hot deserts?
1. Due to heat
2. formation of the subtropical high-pressure cell.
3. rain shadow effect in the belt of easterly trade
winds.
4. effect of the cold currents off the west coast of the
continents at these latitudes.
5. depositing sands of a desert along its border into
the fertile land.
a) 1,3,4,5 b) 2,3,5
c) 2,3,4,5 d) 1,2,3,4
e) All

277. Consider the following statements Answer : (a)


1. Inland Waterways Authority of India (IWAI) is the Inland waterways authority of India’s headquarters is
statutory authority in charge of the waterways in located in Noida, U.P.
India.
2. The headquarter is located in Mumbai.
Which of the above statements is/are correct?
a) Only 1 b) Only 2
c) Both d) None

96
278. Match the following dams with respect to the Answer : (b)
rivers
1. Bhakra Dam ------------------ i. Narmada
2. Almatti Dam ----------------- ii. Godavari
3. Indira sagar----------------- iii. Chambal
4. Jayakwadi Dam---------------- iv.Krishna
5. Rana Pratap Sagar Dam-------- v.Satlaj
a) 1- ii, 2- v, 3-i, 4- iii ,5-iv
b) 1- v, 2- iv, 3-i, 4- ii ,5-iii
c) 1- iii, 2- i, 3-v, 4- ii , 5-iv
d) 1- ii, 2- i, 3-iv, 4- iii ,5-v

279. Match the pairs of Tiger reserves with state in Answer : (d)
which they are located
1. Sariska ----------------i.Mizoram
2. Dampa ----------ii.Maharashtra
3. Kawal-----------iii. Chhattisgarh
4. Bor--------------iv.Rajasthan
5. Indravati-------v. Telangana
a) 1- ii, 2- v, 3-i, 4- iii ,5-iv
b) 1- v, 2- iv, 3-i, 4- ii ,5-iii
c) 1- iii, 2- i, 3-v, 4- ii , 5-iv
d) 1- iv, 2- i, 3-v, 4- ii ,5-iii

280. Match the pairs of valleys with their respective Answer : (d)
region
1. Mandakini Valley ---------- i. Sikkim
2. Damodar Valley ------------ ii. Kerla
3. Silent Valley ------------- iii. Uttarakhand
4. Valley of Flowers --------- iv. Kedarnath
5. Yumthang Valley ----------- v. Jharkhand
a) 1- ii, 2- v, 3-i, 4- iii, 5-iv
b) 1- v, 2- iv, 3-i, 4- ii, 5-iii
c) 1- iii, 2- i, 3-v, 4- ii , 5-iv
d) 1- iv, 2- v, 3-ii, 4- iii, 5-i

97
281. Consider the following statements Answer : (c)
1. Lut desert is salt desert in Iran It recorded 71°temp which is difficult even for bacteria
2. It is the hottest place on Earth to survive.
Which of the above statements is/ are correct?
a) Only 1
b) Only 2
c) Both
d) None

282. Consider the following statements Answer : (c)


1. Don Juan Pond, Antarctica is the saltiest body of Lake Assal is a saline lake which lies below sea level
water on Earth. making it the lowest point on land in Africa and the third-
2. Lake Assal is saltier than the dead sea lowest land depression on Earth after the Dead Sea and
Sea of Galilee. Because of no outflow from the lake and
Which of the above statements is/are correct?
due to high evaporation it is the most saline in the world
a) Only 1 after Don Juan Pond.
b) Only 2
c) Both
d) None

283. Consider the following statements Answer : (a)


1. The Seven North-East States together account for M.P. has the largest forest cover followed by A.P.,
about one fourth of the total forest covers of the Chattisgarh, Maharashtra, Odisha.
country.
2. Arunachal Pradesh has the largest forest cover
Which of the above statements is/are correct?
a) Only 1
b) Only 2
c) Both
d) None

98
284. Match the pairs Answer : (b)
1. Agasthyamala hills --------- i.Mizoram
2. Lushai Hills --------------- ii. Chhattisgarh
3. Satpura Range -------------- iii. Madhya pradesh
4. Mahadeo Hills -------------- iv. Maharashtra
5. Maikal Hills --------------- v. Kerla
a) 1- ii, 2- v, 3-i, 4- iii ,5-iv
b) 1- v, 2- i, 3-iv, 4- iii ,5-ii
c) 1- iii, 2- i, 3-ii, 4- v , 5-iv
d) 1- ii, 2- i, 3-iv, 4- iii ,5-v

285. Consider the following statements Answer : (b)


1. Majuli is the largest river island in the world. Majuli is the largest river island in India. Where the
2. The island is formed by the Brahmaputra river and largest river island in the world in Bananal island formed
the Kherkutia Xuti, an anabranch of the from the bisection of the Araguaia rive in Brazil. Majuli
Brahmaputra, joined by the Subansiri River. had a total area of 1250 sq. km but having lost
significantly to erosion.
3. The residents of Majuli are the "Mising tribes"
from Arunachal Pradesh
Which of the above statements is/are correct?
a) 1, 2
b) 2, 3
c) 1, 3
d) All

286. Match the pairs Answer : (b)


1. Idukki ----------------- i.Odisha
2. Mettur ----------------- ii. Himachal Pradesh
3. Pandoh Dam ------------- iii.TamilNadu
4. Jalaput Dam ------------ iv.Madhya Pradesh
5. Indira Sagar ----------- v. Kerla
a) 1- ii, 2- v, 3-i, 4- iii ,5-iv
b) 1- v, 2- iii, 3-ii, 4- i ,5-iv
c) 1- iii, 2- i, 3-v, 4- ii , 5-iv
d) 1- ii, 2- i, 3-iv, 4- iii ,5-v

99
287. Consider the following statements related to Answer : (c)
Ganges streams Origin of River
1. Mandakini - Alaknanda at Rudraprayag Bhagirthi from Gaumukh
2. Bhagirthi - Alaknanda at Devprayag Alaknanda from Badrinath
Which of the above statement is/are correct? Mandakini from Kedarnath
a) Only 1
b) Only 2
c) Both
d) None

288. Which of the following pair is/are incorrect? Answer : (c)


1. Kosi : Sorrow of Bengal Kosi is called as sorrow of Bihar & Damodar is called as
2. Damodar: Sorrow of Bihar sorrow of Bengal.
a) Only 1
b) Only 2
c) Both
d) None

289. Consider the following statements Answer : (c)


1. Galapagos Islands are located off the coast of Peru Galapagos island are located off the coast of equator.
2. It is situated on equator
3. The island holds the world's only population of
pink iguanas
Which of the above statements are correct?
a) 1, 2
b) 1, 3
c) 2, 3
d) All

290. Which of the following is incorrect Answer : (b)


a) Peru Current: Cold Kuroshio begins off the east coast of Luzon, Philippines,
b) Kuroshio: Cold Jawan and flows north east ward post Japan. It is hot
water current.
c) Aghulas: hot
d) Canari: cold
e) Somali: Hot

100
291. Which of the following are commercial crops? Answer : (c)
1. Coconut A cash crop is an agricultural crop which is grown for sale
2. Jute to return a profit. Coconut, Rubber – plantation crop.
3. Rubber
4. Vegetable
5. Tobacco
a) 1,2 b) 2,3,4
c) 2,5 d) 1,2,3,5

292. Which of the following statement is correct? Answer : ()


1. Taiu river separates India and Myanmar. Amur River Separates China and Afghanistan.
2. Amur River separates china and Russia
3. Mahakali river separates India and Nepal
4. Naf river separates Bangladesh and Myanmar.
a) 1,2,3 b) 2,3,4
c) 1,3,4 d) All

293. The place of occurrence of an earthquake is called Answer : (a)


a. Focus Earthquake refers to the vibration of the earth due to
b. Epicentre the rapid release of energy. The energy released moves
from the focus which is a source point in all the
c. Fault
directions in the form of waves.
d. None of the above

294. Consider the following statements and identify the Answer : (b)
right ones. The intensity and destruction caused by an earthquake is
i. The intensity of an earthquake is greater over generally greater over soft soil regions. For Eg. The
rocky areas. alluvial soil regions. The intensity of an earthquake overy
rockey terrain is lesser.
ii. The destruction caused by an earthquake is
greater over soft soil regions.
a. i only
b. ii only
c. Both
d. None of the above

101
295. Which of the following are features of a primary Answer : (d)
wave? There are three major erosion of seismic waves.
a. Short wavelength Seismic waves are the pulses of energy generated by
b. High frequency earthquakes and they can pass through the entire
earth.
c. Compression wave
d. All the above

296. The Elastic Rebound Theory is associated with Answer : (a)


a. Earthquakes This theory explains the mode and cause of earthquake.
b. Volcanoes The elastic rebound theory is given by Harry F. Reid.
c. Continental drift
d. Atmospheric pressure

297. The place which experiences the seismic event first Answer : (d)
is called The place of the occurrence of an earthquake or the
a. Focus source point is called as focus. Faults denote weaker
b. Eye zones in the Earth's crust. Epicentre is the place where
the seismic event is first experienced.
c. Fault
d. Epicentre

298. Consider the following statements and identify the Answer : (b)
right ones. P-waves are longitudinal travel through crust, mantel.
i. P waves can travel only through solid crust. They also travel through liquid part of earth’s core. Also,
ii. The speed of P waves is the highest. the speed of the primary waves is highest.
a. i only
b. ii only
c. Both
d. None

102
299. Which of the following waves reach the surface of Answer : (a)
the Earth first? P waves are compression waves. Because the speed of
a. P waves the primary waves is the highest, P waves reaches the
b. S waves surface of the Earth faster than the secondary waves or
the long waves.
c. L waves
d. P or S waves

300. Consider the following statements and identify the Answer : (a)
right ones. S-waves do not travel through the liquid part of core. S
i. Secondary waves are also called as distortional waves are transverse waves that have short wave length
waves. and high frequency.
ii. They travel through both solid and liquid part of
the core.
a. i only
b. ii only
c. both
d. None

301. Which of the following are the features of long Answer : (d)
waves?
a. Long wavelength
b. Low frequency
c. Confined to skin of the Earth's crust
d. All the above

302. Consider the following statements and identify the Answer : (d)
right ones. L-waves have low frequency and long wavelength. They
i. L waves have high frequency cover the longest distance than all other seismic waves.
ii. L waves cover the shortest distance of all seismic Long waves are long period waves.
waves.
a. i only
b. ii only
c. Both
d. None

103
303. Consider the following statements and identify the Answer : (c)
right ones. According to continental drift theory, the super
i. Alfred Wegener propounded the continental drift continent called PANGEA started breaking and drifting
theory. away leading to present Continental setup.
ii. It is also known as the displacement hypothesis.

a. i only
b. ii only
c. both
d. None

304. Which of the following is/are the feature/s of a Answer : (d)


tsunami? The term Tsunami is a Japanese word used to mean a
a. high wavelength harbour wave. They are large ocean waves that are a
b. high velocity resultant of large undersea disturbances. That is , it is an
invasion of water on the land because of the dislocation
c. generated by undersea disturbances
in the sea floor.
d. all the above

305. The largest plate identified in the lithosphere is Answer : (a)


a. Pacific plate The plate tectonics theory was forth in 1969. It has 2
b. Juan De Fuca plate components geometric part and kinematic part. The
geometric part is divided into 7 major plates and 12
c. Nazca plate
minor plates.
d. Philippines plate

306. The smallest plate in the lithosphere is Answer : (b)


a. Pacific plate According to the plate tectonics theory given in 1969,
b. Juan De Fuca plate oceans and continents are non-permanent. There are
several individual segments called plates in the
c. Nazca plate
lithosphere. The smallest plate is Juan De Fuca plate
d. Philippines plate which is off the western Coast of North America.

104
307. Consider the following statements and identify the Answer : (c)
right ones. According to the plate tectonics theory, the boundaries
i. A divergent plate boundary results when plates that can be formed as a result of the movement of the
move apart. plates can either be convergent or divergent or
ii. A convergent plate boundary occurs when two transform boundary. When a collision occurs, it can be
either between a oceanic plate and a continental plate or
plates collide.
between 2 oceanic plates or 2 continental plates.
a. i only
b. ii only
c. Both
d. None

308. Consider the following statements and identify the Answer : (c)
right ones. A transform fault is a resultant of two plates sliding each
i. The relative motion of North American plate other. Without converging or delivering from each other
occurs along the San Andreas Fault. San Andreas fault is one of the most seismically active
ii. Transform fault is located where plates slide one faults of the world.
another.
a. i only
b. ii only
c. Both
d. None
309. The concept of sea floor spreading was given Answer : (a)
by
The concept of sea floor spreading was given by Mary
a. Hary Hess
Hess in the year 1960. He wrote, a book titled “An essay
b. Alfred Wegener
in Geopoetry”. He argued that ocean bottom is also
c. F B Taylor permanent and is recycled and its shape is changed.
d. Immanual Kant

310. Consider the following statements and Answer : (a)


identify the right ones.
Mercalli scale is used to measure the intensity or
i. Based on the Mercalli scale, India is divided
destruction caused by the occurrence of earthquake
into 5 seismic zones.
Zone covers intensity of 5 or below, Zone 2 covers 6,
ii. Zone 1 is the highest seismic prone zone. zone 3 covers 7. Accordingly Zone 1 is the least seismic
a. I only prone zone.
b. ii only
c. Both
d. None

311. Identify the regions that come under the Answer : (d)
seismic zone 5.
105
a. The Rann of Kutch Based on the Mercalli scale, the zone 5 records
b. The North East intensity of 9 and abobe , hence these regions are the
c. The Andamans highest seismic prone zone. The Richter Scale
d. All the above measures the magnitude of an earthquake. This zone
has an approximate magnitude of 8 or more on the
Richter scale.

312. Consider the following statements and Answer : (d)


identify the right ones.
There are three major types of volcanoes- the
i. Shield volcanoes often form the tallest
composite, cinder cone and shield volcanoes. Off all
volcanoes.
the three, composite volcanoes often form the tallest
ii. Cinder cone volcanoes are the most
and largest volcanoes. They are also more dangerous
explosive volcanoes.
and explosive than all other volcanoes.
a. I only
b. ii only
c. Both
d. None

313. Match the following. Answer : (a)


These are three major types of volcanoes. A composite
1. Composite volcano ---------- A. Mt. Saint
volcano erupted at Saint Helens in Washington, in
Helens
1980. Paricutin is in Mexico and Muana Loa is in
2. Cinder Cone volcano -------- B. Paricutin
Hawaii. Of all the 3, composite volcano is more
3. Shield volcano ------------- C. Muana Loa
dangerous and a shield volcano has much smaller
eruptions.
a. 1A, 2B, 3C
b. 1B, 2A, 3C
c. 2A, 3B, 3C
d. 1B, 2C, 3A

314. Match the following. Answer : (a)


A volcano refers to an opening or an outburst in the
1. Convergent --------- A. Pacific Ring of Fire
surface or crust of a planet. They are found when
2. Divergent ---------- B. Mid- Atlantic Ridge
tectonic plates are converging or diverging from each
3. Hot Spot ----------- C. Hawaiian Islands
other. They are also formed by hot spots.

a. 1A, 2B, 3C
b. 1B, 2A, 3C
c. 2A, 3B, 3C
d. 1B, 2C, 3A

106
315. Consider the following statements and Answer : (c)
identify the right ones.
Composite volcanoes form the largest and tallest of all
i. Barren Island is the only confirmed active
the volcanoes. They are also more dangerous than the
volcano in South Asia.
other volcanoes. The Barren Island is located in the
ii. Cinder Cone Volcanoes are smaller in size
Andaman Sea. The first volcanic eruption here was
than composite volcanoes.
recorded in 1787.

a. I only
b. ii only
c. Both
d. None

316. Who gave the theory of Nebular Hypothesis? Answer : (b)


a. Harry Hess
The Earth, fifth planet in the solar system dates back to
b. Immanuel Kant
4.6 billion years. Nebular Hypothesis is a theory given
c. F B Taylor
by Kant. It was later revised by Laplace. According to
d. Kepler
this hypothesis, planets were a resultant of a cloud
kind of material that is associated with a slow rotating
star.

317. Consider the following statements and Answer : (a)


identify the right ones.
The inner core is solid in nature while the outer core is
i. The inner core is also known as barysphere.
liquid in nature. The inner core has an approximate
ii. The outer core is solid in nature.
temperature of 6000 degree celcius. Above this core,
a. I only
there is a outer liquid core which consists of molten
b. ii only
iron and nickel.
c. both
d. none

318. Asthenosphere is characterised by Answer : (d)


a. Less rigidity
Asthenosphere is a part of the upper mantle. It is a
b. High temperature
plastic kind of layer lying between the inner-outer
c. Low pressure
mantle and the outer-outer mantle. This is also the
d. All the above
main source of magma coming out of volcanic
eruptions.

107
319. The discontinuity between the mantle and Answer : (c)
core is called
The crust and the mantle is separated by the
a. The mohorovicic discontinuity
mohorovicic discontinuity. This is also known as the
b. The moho discontinuity
moho's discontinuity. The discontinuity between the
c. Guttenberg's discontinuity
mantle and the core is called the guttenberg's
d. The Litho discontinuity
discontinuity.

320. The discontinuity between the crust and the Answer : (a)
mantle is given by

a. The mohorovicic discontinuity


b. Guttenberg's discontinuity
c. Litho discontinuity
d. None of the above

321. Consider the following statements and Answer : (d)


identify the right ones.
The period of nearest distance between the moon and
i. The period of nearest distance between the
earth is called perigee. The period of farthest distance
moon and earth is called apogee.
between the moon and earth is called apogee. The
ii. The period of farthest distance between the
nearest distance is 356,000 kms while the farthest
moon and earth is called perigee.
distance is 4,07,000 kms.
a. I only
b. ii only
c. both
d. none

322. Consider the following statements and Answer : (d)


identify the right ones.
The period of nearest distance between the sun and
earth is called perihelion that occurs on January 3rd,
i. The period of nearest distance between the
while the period of farthest distance between the sun
sun and the earth is called aphelion.
and earth is called as aphelion, that occurs on July 4th.
ii. The period of farthest distance between the
sun and earth is called perihelion.

a. I only
b. ii only
c. both
d. none

108
323. Consider the following statements and Answer : (c)
identify the right ones.
The Earth's axis is inclined at an angle of 661/2 degree
i. The day and night are of equal length
which is the reason for different seasons and
throughout the year at equator.
variations of day and night. Had it been perpendicular,
ii. The region that receives maximum heat is
it would have resulted in equal days and nights at all
called torrid zone.
the time throughout the year.

a. I only
b. ii only
c. both
d. none

324. The term equinoxes refers to Answer : (a)


a. Equal nights
Equinoxes means equal nights. The sun lies vertically
b. Equal distance
overhead at the equator on March 21st and 23rd of
c. Equal pressure
September. On these two days, all places on the Earth
d. Equal angles
have equal days and equal nights.

325. Consider the following statements and Answer : (d)


identify the right ones.
After the March equinox, the sun moves northwards
i. After the March equinox, the sun moves
and on 21st of June, it lies vertically overhead at the
southwards.
tropic of cancer. This is called the summer solstice.
ii. On the summer solstice, northern
Consequently, the northern hemisphere experiences
hemisphere will have the shortest day.
the longest day and shortest night.
a. I only
b. ii only
c. both
d. none

326. Consider the following statements and Answer : (c)


identify the right ones.
After the March equinox, the sun moves northwards
i. After the March equinox, the sun moves
and on 21st of june, it lies vertically overhead at the
northwards.
tropic of cancer. This is called the summer solstice.
ii. On the summer solstice, southern
Consequently, southern hemisphere will have its
hemisphere will have the shortest day.
shortest day and longest night.
a. I only
b. ii only
c. both
d. none

109
327. Consider the following statements and Answer : (b)
identify the right ones.
The sun is overhead at the tropic of capricorn on the
i. The sun is overhead at the tropic of capricorn
22nd of December. Consequently, the southern
on June 21st.
hemisphere has its longest day and shortest night. The
ii. On the winter solstice, southern hemisphere
winter solstice also marks the beginning of summer in
has its longest day.
southern hemisphere and winter in northern
a. I only
hemisphere.
b. ii only
c. both
d. none

328. Consider the following statements and Answer : (a)


identify the right ones.
The Earth has its own shape called the geoid. The term
i. The shape of the earth is called geoid.
geoid itself means earth shaped. The oblate shape is
ii. The oblate shape of the earth is caused by
caused by centrifugal force of rotation of the earth.
centripetal force.
a. I only
b. ii only
c. both
d. none

329. Consider the following statements and Answer : (d)


identify the right ones.
Primary waves are longitudinal waves and they vibrate
i. Primary waves vibrate perpendicular to the
in the same direction as that of the motion of energy.
direction of motion of energy.
Secondary waves are transverse waves and vibrate in
ii. Secondary waves vibrate in the same
the a perpendicular direction to the motion of energy.
direction of the motion of energy.
a. I only
b. ii only
c. both
d. none

110
330. Consider the following statements and Answer : (c)
identify the right ones.
Igneous rocks, also known as the primary rocks are
considered as parents of all other rocks. Extrusive
i. Basalt is an example for extrusive igneous
igneous rock forms out of the magma erupting of
rock.
volcano and its solidification on reaching the surface of
ii.Granite is an example of intrusive igneous
the earth. Intrusive rocks are formed by the same
rock.
solidification, but at moderate depths beneath the
surface of the Earth where cooling is slow.
a. i only
b. ii only
c. both
d. none

331. Identify the feature/s of igneous rocks. Answer : (d)


Igneous rocks are massive and have no layers. They
a. hard
can be light coloured or dark depending on the
b. compact
proportion of silica present in it. If it contains high
c. fossil free
proportion of silica, it will be an acid and high
d. all the above
proportion of basic oxides make them appear dark
and denser.

332. Intrusive igneous rocks include Answer : (d)


a. batholiths
The rock bodies of an igneous rock consists of all the
b. sills
above including laccoliths and stocks. While batholiths,
c. dykes
the largest intrusive rock bodies, form the cores of
d. all the above
world's mountain system, batholiths covering smaller
areas are stocks. Laccoliths are formed between layers
of sedimentary rock near the earth's surface. Dykes
are formed by the passage of liquid material through
cracks and sill is formed between two rock layers.

333. Consider the following statements and Answer : (d)


identify the right ones.
Sediments are carried away by various elements like
i. Loess is an example of ice-deposited
water, wind or ice etc. Loess is an example of a wind-
sedimentary rocks.
borne sedimentary rock and till which is an unsorted
ii. Till is an example of fine sand carried by
mixture of clay and boulders is an ice deposited
wind.
sedimentary rock.
a. i only
b. ii only
c. both
d. none

111
334. Identify the feature/s of a metamorphic rock. Answer : (d)
a. hardness
Metamorphic rock as the name suggests involves
b. closely banded
change. This process takes place under the pressure of
c. interlocking of crystals
the rocks as a result of its contact with some hot
d. all the above
igneous material.

335. Consider the following statements and Answer : (b)


identify the right ones.
The formation of metamorphic rock due to the stress
i. The formation of metamorphic rocks due to
of pressure is called as dynamic metamorphism.
pressure is called contact metamorphism.
Thermal metamorphism is also known as contact
ii. The formation of a metamorphic rock as a
metamorphism and results in the formation of a
result of high temperature is called thermal
metamorphic rock due to the high temperature in the
metamorphism.
earth's crust.
a. I only
b. ii only
c. both
d. none

336. Consider the following statements and Answer : (d)


identify the right ones.
Under dynamic metamorphism which is a result of
i. Granite is converted into gneiss under
excess pressure, granite is converted into gneiss. In
thermal metamorphism.
the thermal metamorphism which is a result of high
ii. Sandstone changes into quartzite under
temperature, sandstone changes into quartzite.
dynamic metamorphism.
a. I only
b. ii only
c. both
d. none

337. Consider the following statements and Answer : (a)


identify the right ones.
Clay and shale are transformed into slate under
i. Clay and shale are transformed into slate
thermal metamorphism which is also known as
under thermal metamorphism.
contact metamorphism. clay and shale are
ii. Clay and shale are transformed into schist at
transformed into schist inter dynamic metamorphism.
contact metamorphism.
a. I only
b. ii only
c. both
d. none

112
337. The plateaus that rise abruptly from the Answer : (b)
lowlands are called
Continental plateaus are formed as a result of an
abrupt rise of a plateau from a lowland or a sea. They
a. Piedmont plateaus
are the resultant of a continental uplift. Example for
b. Continental plateaus
this is the Karnataka and Ranchi plateau in India.
c. Lava plateaus
d. Intermontane plateaus

338. The plateaus that are enclosed by mountains Answer : (d)


are called
Intermontane plateaus are enclosed with the
mountains either fully or partly. The best example for
a. Piedmont plateaus
this type of plateaus is the Tibetan plateau which is
b. Continental plateaus
enclosed by the mountains.
c. Lava plateaus
d. Intermontane plateaus

339. The plateaus that are situated at the foot of a Answer : (a)
mountain are called
Piedmont plateaus are situated at the foot of the
a. Piedmont plateaus
mountains. Also, they are bounded by a plain or an
b. Continental plateaus
ocean on the opposite side. The best example of such
c. Lava plateaus
a plateau is the Malwa plateau in India.
d. Intermontane plateaus

340. The delta formed when the river water is Answer : (b)
lighter than the sea water is
When the river water is as dense as the sea water, it is
a. fan-shaped delta
called a lobate or fan-shaped data. delta refers to the
b. bird-foot data
deposition which is mostly a triangular shaped.
c. arcuate delta
Sundarban is the largest delta in the World.
d. cuspate delta

113
341. Match the following. Answer : (a)
1. arcuate delta ------------- A. Indus
The other examples of an arcuate delta are the Nile,
2. Estuarine delta ----------- B. Narmada
3. Cuspate delta ------------- C. Ebro Ganga, Rhine, Niger, Irrawaddy, Volga, Danube,
4. Bird-foot delta ----------- D. Mississippi Mekong etc and river Omo is also an example of a
a. 1A, 2B,3C,4D bird-foot delta. Rivers Tapi, Amazon, Mackenzie,
b. 1A, 2C,3B,4D
Vistuala, Elb etc are examples of estuarine deltas.
c. 1C, 2A,3D,4C
d. 1D, 2C,3B,4A

342. Consider the following statements and Answer : (d)


identify the right ones.
i. Coral reefs require less mean annual The structures that are produced by living organisms,
temperature. coral reefs are found in the shallow waters. The
ii. Corals live in deeper waters. survival of the corals require high mean annual
a. i only
temperature.
b. ii only
c. both
d. none

343. Consider the following statements and Answer : (a)


identify the right ones.
The Great Barrier Reef is located in Queensland of
i. The Great Barrier Reef is the World’s largest
coral system. Australia and the Red Sea Coral Reef in the coast of
ii. The Red Sea coral Reef is the deepest Egypt and Saudi Arabia. The 2nd largest Coral reef
photosynthetic coral reef. system, the Belize Barrier Reef is in Central America.
a. i only
b. ii only
c. both
d. none

344. Which is the deepest photosynthetic coral Answer : (c)


reef?
Pulley Ridge is located in Florida, United States of
a. The Great Barrier Reef
b. The Belize Barrier Reef America. It is the deepest photosynthetic coral reef
c. Pulley Ridge system in the world.
d. Red Sea Coral Reef

114
345. Identify the factors that contribute to the Answer : (d)
coral bleaching.
Coral bleaching leads to loss of colour of the corals
a. pathogen infections
b. change in salinity level due to stress caused by various factors like global
c. increased sedimentation warming, increase in the solar irradiance, increased
d. all the above sedimentation etc.

346. Consider the following statements and Answer : (d)


identify the right ones.
The opposite line of Greenwich Meridian is the
i. One day is gained as we cross from western
hemisphere to eastern hemisphere. International Date Line. One day is gained as we cross
ii. One day is lost as we cross from eastern to from eastern hemisphere to western hemisphere and
western hemisphere. one day is lost as we cross from western hemisphere
a. i only
to eastern hemisphere.
b. ii only
c. both
d. none

347. Which of the following affects atmospheric Answer : (d)


pressure?
The pressure exerted by air on the surface of the earth
a) Altitude
b) Temperature is called as atmospheric pressure. The pressure varies
c) Earth rotation from region to region.
d) All the above

348 Consider the following statements and identify Answer : (b)


the right ones.
Air pressure increases when air descends. Air pressure
i) Air pressure decreases when air descends.
ii) Air pressure at sea level is lower than at the at sea level is higher than at the mountain top.
mountain top.
a. i only
b. ii only
c. Both
d. None

115
349 Consider the following statements and identify Answer : (d)
the right ones.
The temperature of air falls when its pressure falls and
i )The temperature of air rises when its
pressure falls. the pressure of the air rises when the temperature
ii )The pressure of the air falls when the falls.
temperature rises.
a. i only
b. ii only
c. Both
d. None

350. Consider the following statements and Answer : (d)


identify the right ones.
Low temperature at poles cause the air to contract.
i )Low temperature at poles cause air to
expand. High temperatures along the equator causes air to
ii ) High temperatures along equator cause air expand.
to contract.
a. i only
b. ii only
c. Both
d. None

351. Consider the following statements and Answer : (d)


identify the right ones.
Lower layers of atmosphere have high pressure
i ) Lower layers of atmosphere have low
pressure. because the density is greatest at lower layers and are
ii ) higher layers of atmosphere have high compressed. Higher layers of atmosphere have low
pressure. pressure because they are less compressed.
a. i only
b. ii only
c. Both
d. None

116
352. Consider the following statements and Answer : (d)
identify the right ones. The gradual dissipation of the frontal zone is called
i )The gradual dissipation of the frontal zone is frontolysis. The process by which two air masses of
called frontogensis. different physical characteristics are brought together
ii) The process by which two air masses of is Frontogensis and it leads to the formation of a
different physical characteristics are brought temperate cyclone.
together is frontolysis.
a. i only
b. ii only
c. Both
d. None

353. The weight of water vapour per unit weight of Answer : (a)
air is Specific humidity is expressed as grams per kg of air.
a) Specific Humidity The temperature at which saturation occurs is called
b) Relative humidity dew point. The weight of water vapour per unit weight
c) Absolute Humidity of air is called specific humidity.
d) None of the above

354. The weight of actual amount of water vapour Answer : (c)


present in a unit volume as air is _______ Absolute humodity is expressed as grams per cubic
a) Specific Humidity metre of air. It is the weight of actual amount of water
b) Relative Humidity vapour present in a unit volume as air.
c) Absolute Humidity
d) None of the above

355. The ratio of air's actual water vapour content Answer : (b)
to its water vapour capacity at a given Relative humidity is expressed in terms of percentage.
temperature is called It is the ratio of the actual water vapour content of the
a) Specific Humidity air to its water vapour capacity at a given temperature.
b) Relative Humidity
c) Absolute Humidity
d) None of the above

117

You might also like